Preview (15 of 99 pages)

NCLEX-RN
Part 2
201. A child receiving chemotherapeutic drugs experiences a loss of appetite directly related to the therapy.
Which of the following strategies should be most effective in encouraging the child to eat?
A. Provide a well-balanced diet at usual times, and restrict dessert if the child fails to eat well.
B. Schedule procedures immediately after eating so that the child will not be tired or in pain at mealtime.
C. Offer the child a diet with a wider variety of foods and with more seasoning than her usual diet.
D. Offer the child smaller meals more frequently than usual, and include as many of her favorite foods as
possible.
Answer: D
Explanation:
(A) Because the child's appetite is capricious at best, regular servings may be overwhelming. Praise the
child for what is eaten.
(B) The child will soon learn that procedures follow meals and may play with food rather than eat it to
avoid or delay the procedure.
(C) Young children usually do not like highly seasoned foods and may need the security of usual foods.
Such a change may actually increase anorexia.
(D) Small servings appear more achievable to the child, and the inclusion of favorite foods can add a sense
of security.
202. A 14-year-old teenager is hospitalized for anorexia nervosa. She is admitted to the adolescent mental
health unit and placed on a behavior modification program. Nursing interventions for the teenager will
most likely include:
A. Establishing routine tasks and activities around mealtimes
B. Administering medications such as lithium
C. Requiring the client to eat more during meals
D. Checking the client's room frequently
Answer: A
Explanation:
(A) Providing a more structured, supportive environment addresses safety and comfort needs, thereby
helping the anorexic client develop more internal control.

(B) Medications (commonly antidepressants) are frequently ordered for the anorexic client. However,
lithium (used primarily with bipolar disorder) is not commonly used to treat the anorexic client.
(C) Requiring and/or demanding that the anorexic client "eat more" at mealtimes increases the client's
feelings of powerlessness.
(D) Like the previous strategy, checking the client's room frequently contributes to the client's feelings of
powerlessness.
203. The nurse has been assigned a client who delivered a 6- lb, 12-oz baby boy vaginally 40 minutes ago.
The initial assessment of greatest importance for this client would be:
A. Length of her labor
B. Type of episiotomy
C. Amount of IV fluid to be infused
D. Character of the fundus
Answer: D
Explanation:
The length of labor has little bearing on the fourth stage of labor. The type of labor and delivery is
significant.
(B) The type of episiotomy will affect the client's comfort level. However, the nurse's assessment and
implementations center on prevention of haemorrhage during the fourth stage of labor. The amount of
bleeding from the episiotomy or hematoma formation is of higher priority than the type of episiotomy.
(C) The amount of IV fluid to be infused is a nursing function to be attended to; however, it is lower in
priority than determining if haemorrhaging is occurring.
(D) Character of the fundus would be the priority nursing assessment because changes in uterine tone may
identify possible postpartum haemorrhage.
204. A 5-year-old child has suffered second-degree thermal burns over 30% of her body. Fortyeight hours
after the burn injury, the nurse must begin to monitor the child for which one of the following
complications?
A. Fluid volume deficit
B. Fluid volume excess
C. Decreased cardiac output
D. Severe hypotension
Answer: B

Explanation:
(A) Fluid volume deficit resulting from fluid shifts to the interstitial spaces occurs in the first 48 hours.
(B) Forty-eight hours to 72 hours after the burn injury and fluid resuscitation, capillary permeability is
restored and fluid requirements decrease. Interstitial fluid returns rapidly to the vascular compartment, and
the nurse must monitor the child for signs and symptoms of hypervolemia.
(C) Increased cardiac output results as fluids shift back to the vascular compartment.
(D) Hypertension is the result of hypervolemia.
205. A schizophrenic is admitted to the psychiatric unit. What affect would the nurse expect to observe?
A. Anger
B. Apathy and flatness
C. Smiling
D. Hostility
Answer: B
Explanation:
(A) Anger is an emotion that is not necessarily present in schizophrenia.
(B) Lack of response to or involvement with environment and distancing are characteristic of
schizophrenia.
(C) Euphoria is more characteristic of manic-depressive disorder (bipolar disorder).
(D) Hostility is an emotion that is not necessarily present in schizophrenia.
206. A primigravida with a blood type A negative is at 28 weeks' gestation. Today her physician has
ordered a RhoGAM injection. Which statement by the client demonstrates that more teaching is needed
related to this therapy?
A. "I'm getting this shot so that my baby won't develop antibodies against my blood, right?"
B. "I understand that if my baby is Rh positive I'll be getting another one of these injections."
C. "This shot should help to protect me in future pregnancies if this baby is Rh positive, like my husband."
D. "This shot will prevent me from becoming sensitized to Rh-positive blood."
Answer: A
Explanation:
(A) RhoGAM is given to Rh-negative mothers to prevent the maternal Rh immune response to fetal Rhpositive antigens.

(B) If the infant is Rh positive, the mother will receive another dose postdelivery to prevent maternal
sensitization.
(C) Prevention of maternal sensitization will protect future pregnancies because the mother's blood will be
free of antibodies against her fetus.
(D) RhoGAM prevents maternal sensitization to Rh-positive blood.
207. A client who has gout is most likely to form which type of renal calculi?
A. Struvite stones
B. Staghorn calculi
C. Uric acid stones
D. Calcium stones
Answer: C
Explanation:
(A) The presence of urinary tract infection is a factor in the formation of struvite stones.
(B) Staghorn calculi is the other name for struvite stones associated with urinary tract infection.
(C) Clients who have gout form uric acid stones.
(D) Clients who have increased urinary excretion of calcium form calcium stones.
208. A 55-year-old man is admitted to the hospital with complaints of fatigue, jaundice, anorexia, and claycolored stools. His admitting diagnosis is "rule out hepatitis." Laboratory studies reveal elevated liver
enzymes and bilirubin. In obtaining his health history, the nurse should assess his potential for exposure to
hepatitis. Which of the following represents a high-risk group for contracting this disease?
A. Heterosexual males
B. Oncology nurses
C. American Indians
D. Jehovah's Witnesses
Answer: B
Explanation:
(A) Homosexual males, not heterosexual males, are at high risk for contracting hepatitis.
(B) Oncology nurses are employed in high-risk areas and perform invasive procedures that expose them to
potential sources of infection.
(C) The literature does not support the idea that any ethnic groups are at higher risk.
(D) There is no evidence that any religious groups are at higher risk.

209. A client is to be discharged from the hospital and is to continue taking warfarin 2.5 mg po bid. Which
of the following should be included in her discharge teaching concerning the warfarin therapy?
A. "If you forget to take your morning dose, double the night time dose."
B. "You should take aspirin instead of acetaminophen (Tylenol) for headaches."
C. "Carry a medications alert card with you at all times."
D. "You should use a straight-edge razor when shaving your arms and legs."
Answer: C
Explanation:
(A) Warfarin must always be taken exactly as directed. Clients should be instructed never to skip or double
up on their dosage.
(B) Aspirin decreases platelet aggregation, which would potentiate the effects of the coumadin.
(C) Healthcare providers need to be aware of persons on warfarin therapy prior to the initiation of any
diagnostic tests and/or surgery to help prevent bleeding complications.
(D) An electric razor should be used to prevent accidental cutting, which can lead to bleeding.
210. A 2-year-old boy fell out of bed and has a subdural hematoma. When his mother leaves him for the
first time, you will expect the child to:
A. Be comforted when he is held
B. Cry
C. Not notice that his mother has left
D. Withdraw and become listless
Answer: B
Explanation:
(A) It will be difficult to comfort a 2 year old with a headache without his mother.
(B) This baby probably will cry, which should be prevented because it will increase his intracranial
pressure (ICP). Asking the mother to wait until the baby is asleep may help.
(C) An awake 2 year old will notice when his mother leaves.
(D) An older child may withdraw when feeling afraid, but a 2 year old will probably show more aggressive
behavior.
211. A laboring client presents with a prolapsed cord. The nurse should immediately place the client in
what position?

A. Reverse Trendelenburg
B. Fowler's
C. Trendelenburg
D. Sims'
Answer: C
Explanation:
(A) Reverse Trendelenburg position increases pressure on the perineum. This position will not relieve cord
pressure.
(B) Fowler's position increases perineal pressure. Cord pressure would not be relieved.
(C) Trendelenburg position will decrease perineal pressure. Cord compression will be decreased and
increase in fetal blood flow occurs.
(D) Sims' position does not relieve pressure on cord or perineum.
212. Parents of children receiving chemotherapy should be warned that alopecia is a side effect and that:
A. Children seldom show concern about losing their hair
B. The hair will come out gradually, and the loss will not be noticeable for some time
C. It is best for girls to choose a wig similar to their hair style and color before the hair falls out
D. The parents will soon get used to seeing their children without hair, and it will no longer bother them
Answer: C
Explanation:
(A) Children may become depressed with a changed appearance and not want to look at themselves or
have others see them.
(B) The hair will fall out in clumps, causing patchy baldness that is quite noticeable and traumatic to
children and their families.
(C) Having a wig that looks like a girl's own hair can be a psychological boost to children and is helpful in
fostering later adjustments to hair loss.
(D) Families may become accustomed to seeing their children without hair, but the loss is traumatic to
them and will continue to bother them.
213. A 16-year-old client comes to the prenatal clinic for her monthly appointment. She has gained 14 lb
from her 7th to 8th month; her face and hands indicate edema. She is diagnosed as having PIH and referred
to the high-risk prenatal clinic. The client's weight increase is most likely due to:
A. Overeating and subsequent obesity

B. Obesity prior to conception
C. Hypertension due to kidney lesions
D. Fluid retention
Answer: D
Explanation:
(A) Overeating can lead to obesity, but not to edema.
(B) There is no indication of obesity prior to pregnancy. PIH is more prevalent in the underweight than in
the obese in this age group.
(C) Hypertension can be due to kidney lesions, but it would have been apparent earlier in the pregnancy.
(D) The weight gain in PIH is due to the retention of sodium ions and fluid and is one of the three cardinal
symptoms of PIH.
214. The nurse is caring for a 3-month-old girl with meningitis. She has a positive Kernig's sign. The nurse
expects her to react to discomfort if she:
A. Dorsiflexes her ankle
B. Flexes her spine
C. Plantiflexes her wrist
D. Turns her head to the side
Answer: B
Explanation:
(A) Discomfort with ankle dorsiflexion is not expected with meningitis.
(B) Spinal flexion, flexing the neck or the hips with legs extended, causes discomfort if the meninges are
irritated.
(C) Discomfort with wrist flexion is not expected with meningitis.
(D) Rotating the cervical spine may cause discomfort with meningitis, but pain with flexion is more
indicative of meningeal irritation.
215. A client suspects that she is pregnant. She reports two missed menstrual periods. The first day of her
last menstrual period was August 3. Her estimated date of confinement would be:
A. November 7
B. November 10
C. May 7
D. May 10

Answer: D
Explanation:
(A) Wrong calculation
(B) Wrong calculation
(C) Wrong calculation
(D) Nagele's rule is: Expected Date of Confinement = Last Menstrual Period - 3 months + 7 days + 1 year
216. A 10-year-old boy has been diagnosed with Legg-Calve Perthes disease. Which of the client's
responses would indicate compliance during initial therapy?
A. Drinking large amounts of milk
B. Not bearing weight on affected extremity
C. Walking short distances 3 times/day
D. Putting self on weight reduction diet
Answer: B
Explanation:
(A) This condition causes aseptic necrosis of the head of the femur in the acetabulum. Drinking large
quantities of milk at this time cannot hasten recovery.
(B) The aim of treatment is to keep the head of the femur in the acetabulum. Non-weight-bearing is
essential. Activity causes microfractures of the epiphysis.
(C) In addition to non-weightbearing, clients are often placed on bedrest, which helps to reduce
inflammation. Later, active motion is encouraged.
(D) Weight is not generally an issue with this disease. Slipped femoral capital epiphysis, which is most
frequently observed in obese pubescent children, usually requires a weight reduction diet.
217. A 20-year-old female client delivers a stillborn infant. Following the delivery, an appropriate response
by the labor nurse to the question, "Why did this happen to my baby?" is:
A. "It's God's will. It was probably for the best. There was something probably wrong with your baby."
B. "You're young. You can have other children later."
C. "I know your other children will be a great comfort to you."
D. "I can see you're upset. Would you like to see and hold your baby?"
Answer: D
Explanation:
(A) The mother and the father require support; the nurse should not minimize their grief in this situation.

(B) Attachment to this infant occurs during the pregnancy for both the mother and father. Another child
cannot replace this child.
(C) Attachment to this infant occurs during the pregnancy for both the mother and father. Siblings will not
replace their feelings or minimize their loss of this infant.
(D) Holding and viewing the infant decreases denial and may facilitate the grief process. The nurse should
prepare family members for how the infant appears ("she is bruised") and provide support.
218. A 16-year-old client with anorexia nervosa is on an inpatient psychiatric unit. She has a fear of
gaining weight and is refusing to eat sufficient amounts to maintain body weight for her age, height, and
stature. To assist with the problem of powerlessness and plan for the client to no longer need to withhold
food to feel in control, the nurse uses the following strategy:
A. Establish a structured environment with routine tasks and activities. Also, serve meals at the same time
each day.
B. Distract the client during meals to get her to eat because she must take in sufficient amounts to keep
from starving.
C. Do frequent room checks to be sure that the client is not hiding food or throwing it away.
D. Listen attentively and participate in in-depth discussions about food, because these actions may
encourage her to eat.
Answer: A
Explanation:
(A) Anorexia nervosa clients feel out of control. Providing a structured environment offers safety and
comfort and can help them to develop internal control, thus reducing their need to control by self
starvation.
(B) Distraction does not focus on the client's need for control.
(C) Doing frequent room checks reinforces feelings of powerlessness and the need to continue with the
dysfunctional behavior.
(D) Participating in long discussions about food does not make the client want to eat, but rather this
strategy allows her to indulge in her preoccupation and to continue with the dysfunctional behavior.
219. The nurse is caring for a 2-year-old girl with a subdural hematoma of the temporal area as a result of
falling out of bed and notices that she has a runny nose. The nurse should:
A. Call the doctor immediately
B. Help her to blow her nose carefully

C. Test the discharge for sugar
D. Turn her to her side
Answer: C
Explanation:
(A) The nasal discharge could be due to a cold. It is necessary to gather additional assessment data to
identify a possible cerebrospinal fluid leak.
(B) If the discharge is cerebrospinal fluid, it would not be safe to encourage the girl to blow her nose.
(C) Cerebrospinal fluid is positive for sugar; mucus is not.
(D) Turning her to her side will have no effect on her "runny nose." It is necessary to gather further
assessment data.
220. Which of the following menu choices would indicate that a client with pressure ulcers understands the
role diet plays in restoring her albumin levels?
A. Broiled fish with rice
B. Bran flakes with fresh peaches
C. Lasagna with garlic bread
D. Cauliflower and lettuce salad
Answer: A
Explanation:
(A) Broiled fish and rice are both excellent sources of protein.
(B) Fresh fruits are not a good source of protein.
(C) Foods in the bread group are not high in protein.
(D) Most vegetables are not high in protein; peas and beans are the major vegetables higher in protein.
221. The nurse would be sure to instruct a client on the signs and symptoms of an eye infection and
haemorrhage. These signs and symptoms would include:
A. Blurred vision and dizziness
B. Eye pain and itching
C. Feeling of eye pressure and headache
D. Eye discharge and haemoptysis
Answer: B
Explanation:

(A) Although blurred vision may occur, dizziness would not be associated with an infection or
haemorrhage.
(B) Eye pain is a symptom of haemorrhage within the eye, and itching is associated with infection.
(C) Nausea and headache would not be usual symptoms of eye haemorrhage or infection.
(D) Some eye discharge might be anticipated if an infection is present; haemoptysis would not.
222. A client has ascites, which is caused by:
A. Decreased plasma proteins
B. Electrolyte imbalance
C. Decreased renal function
D. Portal hypertension
Answer: A
Explanation:
(A) A decrease in plasma proteins causes a decrease in intravascular osmotic pressure resulting in leakage
of fluid into peritoneal cavity.
(B) Fluid and electrolyte imbalance may occur as a result of the ascites.
(C) Ascites is a result of hepatic malfunction, not renal malfunction.
(D) Portal hypertension causes esophageal varices, not ascites.
223. A 19-year-old primigravida is admitted to the labor and delivery suite of the hospital. Her husband is
accompanying her. The couple tells the nurse that this is the first hospital admission for her. The client's
vaginal exam indicates she is 3 cm dilated, 80% effaced, and at _0 station. Based on the vaginal exam, she
is in:
A. Stage 2, latent phase
B. Stage 1, active phase
C. Stage 3, transition phase
D. Stage 1, latent phase
Answer: D
Explanation:
(A) The second stage of labor is from full cervical dilation through birth of the baby. The three phases of
this stage include latency or resting, descent, and final transition. The client is less than fully dilated so she
is not in stage 2.

(B) The first stage of labor begins with regular uterine contractions and continues until the woman is 10
cm dilated. The three phases of this stage include the early or latent phase (0-3 cm), the active phase (4-7
cm), and the transitional phase (7-10 cm). The client is 40%
D. Maintain on 100% O2
Answer: A
Explanation:
(A) Retrolental fibroplasia is the result of prolonged exposure to high levels of O2 in premature infants.
Complications are haemorrhage and retinal detachment.
(B, C, D) O2 concentration is too high.
293. A 6-year-old girl has been diagnosed with a urinary tract infection secondary to vesicoureteral reflux.
Which statement by her mother indicates a need for further teaching?
A. "I have taught her to wipe from front to back after urinating."
B. "I make sure she drinks plenty of fluids every day."
C. "She enjoys wearing nylon panties, but I make her change them everyday."
D. "She tries to empty her bladder completely after she urinates, like I told her."
Answer: C
Explanation:

(A) Wiping from front to back is wiping from an area of lesser contamination (urethra) to an area of
greater contamination (rectum).
(B) Generous fluid intake reduces the concentration of urine.
(C) Cotton is a natural, absorbent fabric. Nylon often predisposes the client to urinary tract infections.
Dark, warm, moist areas are excellent media for bacterial growth.
(D) With vesicoureteral reflux, urine refluxes into the ureter(s) during voiding and then returns to the
bladder (residual), which becomes a source for future infection.
294. A neonate was admitted to the hospital with projectile vomiting. According to the parents, the baby had
experienced vomiting episodes after feeding for the last 2 days. A medical diagnosis of hypertrophic pyloric
stenosis was made. On assessment, the infant had poor skin turgor, sunken eyeballs, dry skin, and weight
loss. Identify the number-one priority nursing diagnosis.
A. Fluid volume deficit
B. Altered nutrition
C. Altered bowel elimination
D. Anxiety
Answer: A
Explanation:
(A) Fluid volume deficit is the major problem. Symptoms of dehydration are evident. The effects of fluid
and electrolyte balance may be life threatening. Rehydration can be accomplished effectively through IV
fluids and electrolytes.
(B) Vomiting may also signal a nutritional problem. However, the nutritional problem would be secondary
to fluid and electrolyte disturbances. The infant may also be placed on NPO status.
(C) With vomiting, a decrease in the size and number of stools is expected.
(D) The infant cannot verbalize feelings of anxiety. Anxiety would not be an appropriate diagnosis.
295. The nurse is teaching a 10-year-old insulin-dependent diabetic how to administer insulin.
Which one of the following steps must be taught for insulin administration?
A. Never use abdominal site for a rotation site.
B. Pinch the skin up to form a subcutaneous pocket.
C. Avoid applying pressure after injection.
D. Change needles after injection.
Answer: B

Explanation:
(A) Sites for injection need to be rotated, including abdominal sites, to enhance insulin absorption.
(B) The pinch technique is the most effective method for obtaining skin tightness to allow easy entrance of
the needle to subcutaneous tissues.
(C) Massaging the site of injection facilitates absorption of the insulin.
(D) Changing the needle will break the sterility of the system. It has become acceptable practice to reuse
disposable needles and syringes for 3-7 days.
296. The health team needs to realize that the compulsive concern with cleanliness that a client with severe
anxiety exhibits is most likely an attempt to:
A. Reduce his anxiety
B. Avoid going to psychotherapy
C. Manipulate the health team members
D. Increase his self-image by showing higher standards than the fellow clients
Answer: A
Explanation:
(A) These behaviors are attempts to relieve anxiety.
(B) Avoidance is not a pattern in the obsessive client.
(C) Although these behaviors may seem to manipulate others, that is not the purpose behind the activity.
(D) Inflated self-esteem is not a characteristic of the severely anxious client.
297. A physician's order reads: Administer furosemide oral solution 0.5 mL stat. The furosemide bottle
dosage is 10 mg/mL. What dosage of furosemide should the nurse give to this infant?
A. 5 mg
B. 0.5 mg
C. 0.05 mg
D. 20 mg
Answer: A
Explanation:
(A) 1 mg = 0.1 mL, then 0.5 mL X= 55 mg.
(B) Thisanswer is a miscalculation.
(C) This answer is a miscalculation.
(D) This answer is a miscalculation.

298. A client is being discharged with albuterol (Proventil) and beclomethasone dipropionate (Vanceril) to
be administered via inhalation three times a day and at bedtime. Client teaching regarding the sequential
order in which the drugs should be administered includes:
A. Glucocorticoid followed by the bronchodilator
B. Bronchodilator followed by the glucocorticoid
C. Alternate successive administrations
D. According to the client's preference
Answer: B
Explanation:
(A) The client would not receive therapeutic effects of the glucocorticoid when it is inhaled through
constricted airways.
(B) Broncho dilating the airways first allows for the glucocorticoid to be inhaled through open airways and
increases the penetration of the steroid for maximum effectiveness of the drug.
(C) Inac-Inaccurate use of the inhalers will lead to decreased effectiveness of the treatment.
(D) Client teaching regarding the use and effects of inhalers will promote client understanding and
compliance.
299. The nurse assists a client with advanced emphysema to the bathroom. The client becomes extremely
short of breath while returning to bed. The nurse should:
A. Increase his nasal O2 to 6 L/min
B. Place him in a lateral Sims' position
C. Encourage pursed-lip breathing
D. Have him breathe into a paper bag
Answer: C
Explanation:
(A) Giving too high a concentration of O2 to a client with emphysema may remove his stimulus to breathe.
(B) The client should sit forward with his hands on his knees or an overbed table and with shoulders
elevated.
(C) Pursed-lip breathing helps the client to blow off CO2 and to keep air passages open.
(D) Covering the face of a client extremely short of breath may cause anxiety and further increase dyspnea.

300. A 27-year-old man was diagnosed with type I diabetes 3 months ago. Two weeks ago he complained
of pain, redness, and tenderness in his right lower leg. He is admitted to the hospital with a slight elevation
of temperature and vague complaints of "not feeling well." At 4:30
PM on the day of his admission, his blood glucose level is 50 mg; dinner will be served at 5:00 PM.
The best nursing action would be to:
A. Give him 3 tbsp of sugar dissolved in 4 oz of grape juice to drink
B. Ask him to dissolve three pieces of hard candy in his mouth
C. Have him drink 4 oz of orange juice
D. Monitor him closely until dinner arrives
Answer: C
Explanation:
(A) The combination of sugar and juice will increase the blood sugar beyond the normal range.
(B) Concentrated sweets are not absorbed as fast as juice; consequently, they elevate the blood sugar
beyond the normal limit.
(C) Four ounces of orange juice will act immediately to raise the blood sugar to a normal level and sustain
it for 30 minutes until supper is served.
(D) There is an increased potential for the client's blood sugar to decrease even further, resulting in
diabetic coma.
301. A client presents to the psychiatric unit crying hysterically. She is diagnosed with severe anxiety
disorder. The first nursing action is to:
A. Demand that she relax
B. Ask what is the problem
C. Stand or sit next to her
D. Give her something to do
Answer: C
Explanation:
(A) This nursing action is too controlling and authoritative. It could increase the client's anxiety level.
(B) In her anxiety state, the client cannot rationally identify a problem.
(C) This nursing action conveys a message of caring and security.
(D) Giving the client a task would increase her anxiety. This would be a late nursing action.

302. A client is in active labor and has been admitted to the labor and delivery unit. The RN has just done a
sterile vaginal exam and determines that the client is dilated 5 cm, effaced 85%, and the fetus's head is at 0
station. She asks if she could have a lumbar epidural now. The epidural is started, and the anesthetic agent
used is bupivacaine (Marcaine). After the client has received her lumbar epidural, it is important for the
RN to monitor her for which of the following side effects:
A. Hypertension
B. Hypotension
C. Hypoglycemia
D. Hyperglycemia
Answer: B
Explanation:
(A) The medication bupivacaine will cause vasodilation in the vascular system, and this does not result in
elevation of the ma-ternal blood pressure.
(B) The medication bupivacaine will cause vasodilation in the vascular system, and this will result in
lowering the maternal blood pressure.
(C) Bupivacaine does not interfere with the functioning of the endocrine system.
(D) Bupivacaine does not interfere with the functioning of the endocrine system.
303. A 35-year-old client is receiving psychopharmacological treatment of his major depression with
tranylcypromine sulfate (Parnate), a monoamine oxidase (MAO) inhibitor. The nurse teaches the client that
while he is taking this type of antidepressant, he needs to restrict his dietary intake of:
A. Potassium-rich foods
B. Tryptophan
C. Tyramine
D. Saturated fats
Answer: C
Explanation:
(A) The client may need to avoid some potassium-rich foods (such as bananas, raisins, etc.). However, this
is not because of the potassium content of these foods.
(B) Tryptophan is an essential amino acid that is present in high concentrations in animal and fish protein.
(C) The client will need to watch his dietary intake of tyramine. Tyramine is a by-product of the
conversion of tyrosine to epinephrine. Tyramine is found in a variety of foods and beverages, ranging from

aged cheese to caffeine drinks. Ingestion of tyramine-rich foods while taking a MAO inhibitor may lead to
an increase in blood pressure and/or a life-threatening hypertensive crisis.
(D) To maintain a healthy lifestyle, restriction of dietary saturated fats is advisable.
304. A male client receives 10 U of regular human insulin SC at 9:00 AM. The nurse would expect peak
action from this injection to occur at:
A. 9:30 AM
B. 10:30 AM
C. 12 noon
D. 4:00 PM
Answer: C
Explanation:
(A) This is too early for peak action to occur.
(B) This is too early for peak action to occur.
(C) Regular insulin peak action occurs 2-4 hours after administration.
(D) This is too late for peak action to occur.
305. A female client who has chronic obstructive pulmonary disease (COPD) has presented in the
emergency department with cough productive of yellow sputum and increasing shortness of breath. On
room air, her blood gases are as follows: pH 7.30 mm Hg, PCO2 60 mm Hg, PO2 55 mm Hg, HCO3 32
mEq/L. These arterial blood gases reflect :
A. Compensated respiratory acidosis
B. Normal blood gases
C. Uncompensated metabolic acidosis
D. Uncompensated respiratory acidosis
Answer: D
Explanation:
(A) In compensated respiratory acidosis, the pH level is normal, the PCO2 level is elevated, and the HCO3
level is elevated. The client's primary alteration is an inability to remove CO2 from the lungs, so over time,
the kidneys increase reabsorption of HCO3 to buffer the CO2.
(B) Normal ranges for arterial blood gases for adults and children are as follows: pH 7.35-7.45, PO2 80100 mm Hg, PCO2 35-45 mm Hg, HCO3 21-28 mEq/L.

(C) In uncompensated metabolic acidosis the pH level is decreased, the PCO2 level is normal, and the
HCO3 level is decreased. The client's primary alteration is an inability to remove excess acid via the
kidneys. The lungs are unable to clear the increased acid.
(D) In uncompensated respiratory acidosis, the pH level is decreased, the PCO2 level is increased, and the
HCO3 level is normal. In a person with long-standing COPD, the HCO3 level will rise gradually over time
to compensate for the gradually increasing PCO2, and the person's pH level will be normal. When a person
with COPD becomes acutely ill, the kidneys do not have time to increase the reabsorption of HCO 3, so the
person's pH level will reflect acidosis even though the HCO3 is elevated.
306. As a nurse works with an adolescent with cystic fibrosis, the nurse begins to notice that he appears
depressed and talks about suicide and feelings of worthlessness. This is an important factor to consider in
planning for his care because:
A. It may be a bid for attention and an indication that more diversionary activity should be planned for him
B. No threat of suicide should be ignored or challenged in any way
C. He needs to be observed carefully for signs that his depression has been relieved
D. He needs to be confronted with his feelings and forced to work through them
Answer: B
Explanation:
(A) Threats of suicide should always be taken seriously.
(B) This client has a life-threatening chronic illness. He may be concerned about dying or he may actually
be contemplating suicide.
(C) Sometimes clients who have made the decision to commit suicide appear to be less depressed.
(D) Forcing him to look at his feelings may cause him to build a defense against the depression with
behavioral or psychosomatic disturbances.
307. As a postoperative cholecystectomy client completes tomorrow's dinner menu, the nurse knows that
one of the following meal choices will best provide the essential vitamin(s) necessary for proper tissue
healing?
A. Liver, white rice, spinach, tossed salad, custard pudding
B. Fish fillet, carrots, mashed potatoes, butterscotch pudding
C. Roast chicken, gelatin with sliced fruit
D. Chicken breast fillet in tomato sauce, potatoes, mustard greens, orange and strawberry slices
Answer: D

Explanation:
(A) This meal choice provides more of the vitamins A, D, and K than of vitamin C.
(B) This meal choice provides more of the vitamins A, B12, and D than of vitamin C.
(C) This meal choice provides more of the vitamins A, B1 (thiamine), niacin, and microminerals than of
vitamin C.
(D) This meal choice provides foods rich in vitamin C, which are essential in tissue healing.
308. A 64-year-old client is admitted to the hospital with benign prostatic hypertrophy (BPH). He has a
history of adult-onset diabetes and hypertension and is scheduled to undergo a resection of the prostate.
When recording his health history, the nurse asks about his chief complaint. The most serious symptom
that may accompany BPH is:
A. Acute urinary retention
B. Hesitancy in starting urination
C. Increased frequency of urination
D. Decreased force of the urinary stream
Answer: A
Explanation:
(A) Acute urinary retention requires urgent medical attention. If measures such as a warm tub bath or
warm tea do not occur after 6 hours, the client should go to the ED for catheterization.
(B, C, D) This choice is a symptom of BPH, but it is not serious or life threatening.
309. A 47-year-old male client is admitted for colon surgery. Intravenous antibiotics are begun 2 hours
prior to surgery. He has no known infection. The rationale for giving antibiotics prior to surgery is to:
A. Provide cathartic action within the colon
B. Reduce the risk of wound infection from anaerobic bacteria
C. Relieve the client's concern regarding possible infection
D. Reduce the risk of intraoperative fever
Answer: B
Explanation:
(A) Cathartic drugs promote evacuation of intestinal contents.
(B) The client undergoing intestinal surgery is at increased risk for infection from large numbers of
anaerobic bacteria that inhabit the intestines. Administering antibiotics prophylactically can reduce the
client's risk for infection.

(C) Antibiotics are indicated in the treatment of infections and have no effect on emotions.
(D) Antipyretics are useful in the treatment of elevated temperatures. Antibiotics would have an effect on
infection, which causes temperature elevation, but would not directly affect such an elevation.
310. A client has just received an epidural block. She is laboring on her right side. The nurse notes that her
blood pressure has dropped from 132/68 to 78/42 mm Hg. The nurse's first action would be to:
A. Call the physician immediately and give dopamine IM
B. Turn her on her left side and recheck her blood pressure in 5 minutes
C. Administer oxytocin (Pitocin) immediately and increase the rate of IV fluids
D. Increase the rate of IV fluids and start O2 by mask
Answer: D
Explanation:
(A) Nursing measures to support fetal oxygenation and promote maternal blood pressure would precede
calling the physician.
(B) Systolic pressures below 100 mm Hg or a reduction in the systolic pressure of >30% necessitate
treatment. Assessing the blood pressure in 5 minutes may allow for further fetal and/or maternal
compromise. Turning the client on her left side will promote uteroplacental perfusion and is appropriate.
(C) Oxytocin (Pitocin) increases the strength of uterine contractions and may cause maternal hypotension;
thus it is an inappropriate drug for use in this clinical situation. IV fluids would be increased to expand the
circulating blood volume and promote increased blood pressure.
(D) Turning the mother to her left lateral side promotes uteroplacental perfusion.
IV fluids are administered to increase the circulating blood volume, and O 2 is administered to promote
fetal oxygenation and decrease the nausea accompanying the hypotension.
311. A client who was started on antipsychotic medication 2 weeks ago is preparing for discharge from the
hospital. Compliance with the medication regimen is important despite the mild side effects encountered.
In order to increase the likelihood of medication compliance, the nurse would:
A. Discuss the disease process and the importance of the medication in prevention of symptoms.
B. Inform the client that additional side effects are to be expected and need not be reported.
C. Discuss the importance of getting blood drawn weekly to determine medication therapeutics.
D. Inform the client to cease taking the medication when all psychotic symptoms have cleared.
Answer: A
Explanation:

(A) This answer is correct. If the client is well informed about what reactions to expect from her
medication, she is more likely to follow the treatment regimen.
(B) This answer is incorrect. There are many side effects that are reversible by medication, and these must
be reported to the nurse or physician. There are also more severe side effects, such as neuroleptic
malignant syndrome, characterized by fever, tachycardia, and diaphoresis, which can be life threatening.
(C) This answer is incorrect. There is no need for weekly blood tests if the drug regimen has been followed
properly.
(D) This answer is incorrect. The client should continue the medication until the physician recommends
any change in the drug regimen. Symptoms will usually reappear if medication is discontinued.
312. A 40-year-old client is admitted to the coronary care unit with chest pain and shortness of breath. The
physician diagnosed an anterior wall myocardial infarction. What tests should the nurse anticipate?
A. Reticulocyte count, creatinine phosphokinase (CPK)
B. Aspartate transaminase, alanine transaminase
C. Sedimentation rate, WBC count
D. Lactic dehydrogenase, CPK
Answer: D
Explanation:
(A) Reticulocyte count measures the number of immature erythrocytes. CPK is an enzyme released from
injured myocardial tissue.
(B) Aspartate transaminase is an enzyme released from injured myocardial tissue. Alanine transaminase is
an enzyme released for general tissue destruction, which is specific for liver injury.
(C) Sedimentation rate is a nonspecific test for inflammation.
(D) Lactic dehydrogenase and CPK are enzymes released from injured myocardial tissue.
313. In working with mental health clients who are prescribed medication that must be taken on a routine
basis, it is important for education to begin when the drug therapy is initiated. One of the first steps in the
teaching process is to:
A. Explain the side effects of the medication
B. Discuss the danger of overmedication
C. Distribute written material to supplement verbal instructions
D. Explore the client's perception regarding medication therapy
Answer: D

Explanation:
(A, B, C) The nurse must first obtain information regarding the client's perception of the medication
regimen.
(D) The first step in the teaching process is to determine the client's perception.
314. A term neonate has experienced no distress at birth and has an Apgar score of 9. Her mother has asked
to breastfeed her following delivery. Immediately after birth, the neonate was most susceptible to heat loss.
The most appropriate intervention to conserve heat loss and promote bonding is to:
A. Place her under the radiant warmer
B. Dry her with blankets
C. Place her to her mother's breast
D. Place her on a heated pad
Answer: C
Explanation:
(A) A radiant warmer maintains an optimal thermal environment by use of a thermal skin sensor taped to
the infant. The warmer limits parental attachment, so, although appropriate, it is not an intervention that
promotes infant attachment.
(B) Warmed blankets prevent heat loss in the neonate by conduction. In addition, tactile stimuli promote
crying and lung expansion. This intervention does not promote attachment, however.
(C) Skin-toskin contact is an effective way to conserve heat after delivery and promotes parental
attachment following birth in the healthy term infant. The first period of reactivity lasts approximately 30
minutes following birth. A strong sucking reflex and an active, awake newborn characterize this period.
(D) Surfaces of objects warmer than the infant promote overheating by conduction, and neonatal
hyperthermia may result.
315. Which of the following activities would be most appropriate during occupational therapy for a client
with bipolar disorder?
A. Playing cards with other clients
B. Working crossword puzzles
C. Playing tennis with a staff member
D. Sewing beads on a leather belt
Answer: C
Explanation:

(A) This activity is too competitive, and the manic client might become abusive toward the other clients.
(B) During mania, the client's attention span is too short to accomplish this task.
(C) This activity uses gross motor skills, eases tension, and expands excess energy. A staff member is
better equipped to interact therapeutically with clients.
(D) This activity requires the use of fine motor skills and is very tedious.
316. A postoperative prostatectomy client is preparing for discharge from the hospital the next morning.
The nurse realizes that additional instructions are necessary when he states:
A. "If I drink 10 to 12 glasses of fluids each day, that will help to prevent any clot formation in my urine."
B. "The isometric exercises will help to strengthen my perineal muscles and help me control my urine."
C. "If I feel as though I have developed a fever, I will take a rectal temperature, which is the most
accurate."
D. "I do not plan to do any heavy lifting until I visit my doctor again."
Answer: C
Explanation:
(A) This is correct health teaching. Drinking 10-12 glasses of clear liquid will help increase urine volumes
and prevent clot formation.
(B) This is correct health teaching. These types of exercises are prescribed by physicians to assist
postprostatectomy clients to strengthen their perineal muscles.
(C) This action is not recommended post-TURP because of the close proximity of the prostate and rectum.
(D) This is correct healthcare teaching. The client should limit walking long distances, lifting heavy
objects, or driving a car until these activities are cleared by the physician at the first office visit.
317. After an infant is delivered by caesarean delivery and placed on the warmer, the RN dries and
assesses the infant. At 1 and 5 minutes after birth, the RN does the Apgar scoring of the infant. The RN
knows that because this infant was delivered by caesarean section, he is at increased risk for having which
one of the following:
A. Cold stress
B. Cyanosis
C. Respiratory distress syndrome
D. Seizures
Answer: C
Explanation:

(A) The infant is placed on the warmer and dried after birth. Cold stress occurs when the infant is not dried
and kept warm.
(B) The fact that this infant was born by caesarean delivery does not place him at a greater risk for
cyanosis than an infant delivered vaginally. Cyanosis occurs when infants cannot oxygenate their blood
after the umbilical cord is severed.
(C) Infants born by caesarean delivery are at a higher risk for developing respiratory distress syndrome
because these infants do not pass through the pelvis, where the chest is compressed and fluid is able to
escape from the lungs.
(D) Caesarean delivered infants are not at greater risk for seizures than infants delivered vaginally.
318. Diabetes mellitus is a disorder that affects 3.1 out of every 1000 children younger than 20 years old. It
is characterized by an absence of, or marked decrease in, circulating insulin. When teaching a newly
diagnosed diabetes client, the nurse includes information on the functions of insulin:
A. Transport of glucose into body cells and storage of glycogen in the liver
B. Glycogenolysis and facilitation of glucose use for energy
C. Glycogenolysis and catabolism
D. Catabolism and hyperglycemia
Answer: A
Explanation:
(A) Lack of insulin causes glycogenolysis, catabolism, and hyperglycemia.
(B) Insulin promotes the conversion of glucose to glycogen for storage and regulates the rate at which
carbohydrates are used by cells for energy.
(C) Insulin is anabolic in nature.
(D) Glucose stimulates protein synthesis within the tissue and inhibits the breakdown of protein into amino
acids.
319. A 3-year-old female client is brought into the paediatric clinic because she limps. She has not been to
the clinic since she was 9 months old. The nurse practitioner describes the limp as a "Trendelenburg gait."
This gait is characteristic of:
A. Scoliosis
B. Dislocated hip
C. Fractured femur
D. Fractured pelvis

Answer: B
Explanation:
(A, C, D) A Trendelenburg gait is not characteristic of any of these disorders.
(B) The downward slant of one hip is a positive sign of dislocation in the weight-bearing hip. If one hip is
dislocated, the child walks with a characteristic limp known as the Trendelenburg gait.
320. Diabetes during pregnancy requires tight metabolic control of glucose levels to prevent perinatal
mortality. When evaluating the pregnant client, the nurse knows the recommended serum glucose range
during pregnancy is:
A. 70 mg/dL and 120 mg/dL
B. 100 mg/dL and 200 mg/dL
C. 40 mg/dL and 130 mg/dL
D. 90 mg/dL and 200 mg/dL
Answer: A
Explanation:
(A) The recommended range is 70-120 mg/dL to reduce the risk of perinatal mortality.
(B, C, D) These levels are not recommended. The higher the blood glucose, the worse the prognosis for the
fetus. Hypoglycemia can also have detrimental effects on the fetus.
321. A 4-year-old boy is brought to the emergency room with bruises on his head, face, arms, and legs. His
mother states that he fell down some steps. The nurse suspects that he may have been physically abused. In
accordance with the law, the nurse must:
A. Tell the physician her concerns
B. Report her suspicions to the authorities
C. Talk to the child's father
D. Confront the child's mother
Answer: B
Explanation:
(A) Although the nurse probably would talk to the physician about these concerns, the nurse is not required
by law to do so.
(B) All healthcare workers are required by the Federal Child Abuse Prevention and Treatment Act of 1974
to report suspected and actual cases of child abuse and/or neglect.

(C) Talking to the child's father may or may not help the child, and the nurse is not required by law to do
so.
(D) Confrontation may not be indicated; the nurse is not required by law to confront the child's mother
with these suspicions.
322. An 18-month-old child has been playing in the garage. His mother brings him to a nurse's home
complaining of his mouth being sore. His lips and mouth are soapy and white, with small ulcerated areas
beginning to form. The child begins to vomit. His pulse is rapid and weak. The nurse suspects that the
child has:
A. Inhaled gasoline fumes
B. Ingested a caustic alkali
C. Eaten construction chalk
D. Lead poisoning
Answer: B
Explanation:
(A, C, D) These agents would not cause ulcerations on mouth and lips.
(B) Strong alkali or acids will cause burns and ulceration son the mucous membranes.
323. Following a gastric resection, a 70-year-old client is admitted to the post anesthesia care unit. He was
extubated prior to leaving the suite. On arrival at the post anesthesia care unit, the nurse should:
A. Check airway, feeling for amount of air exchange noting rate, depth, and quality of respirations
B. Obtain pulse and blood pressure readings noting rate and quality of pulse
C. Reassure the client that his surgery is over and that he is in the recovery room
D. Review physician's orders, administering medications as ordered
Answer: A
Explanation:
(A) Adequate air exchange and tissue oxygenation depend on competent respiratory function. Checking
the airway is the nurse's priority action.
(B) Obtaining the vital signs is an important action, but it is secondary to airway management.
(C) Reorienting a client to time, place, and person after surgery is important, but it is secondary to airway
and vital signs.
(D) Airway management takes precedence over physician's orders unless they specifically relate to airway
management.

324. A 29-year-old client is admitted for a hysterectomy. She has repeatedly told the nurses that she is
worried about having this surgery, has not slept well lately, and is afraid that her husband will not find her
desirable after the surgery. Shortly into the preoperative teaching, she complains of a tightness in her chest,
a feeling of suffocation, light headedness, and tingling in her hands. Her respirations are rapid and deep.
Assessment reveals that the client is:
A. Having a heart attack
B. Wanting attention from the nurses
C. Suffering from complete upper airway obstruction
D. Hyperventilating
Answer: D
Explanation:
(A) Classic symptoms of a heart attack include heaviness or squeezing pain in the chest, pain spreading to
the jaw, neck, and arm. Nausea and vomiting, sweating, and shortness of breath may be present. The client
does not exhibit these symptoms.
(B) Clients suffering from anxiety or fear prior to surgical procedures may develop hyperventilation. This
client is not seeking attention.
(C) Symptoms of complete airway obstruction include not being able to speak, and no airflow between the
nose and mouth. Breath sounds are absent.
(D) Tightness in the chest; a feeling of suffocation; light headedness; tingling in the hands; and rapid, deep
respirations are signs and symptoms of hyperventilation. This is almost always a manifestation of anxiety.
325. The nurse is preparing a 6-year-old child for an IV insertion. Which one of the following statements
by the nurse is appropriate when preparing a child for a potentially painful procedure?
A. "Some say this feels like a pinch or a bug bite. You tell me what it feels like."
B. "This is going to hurt a lot; close your eyes and hold my hand."
C. "This is a terrible procedure, so don't look."
D. "This will hurt only a little; try to be a big boy."
Answer: A
Explanation:
(A) Educating the child about the pain may lessen anxiety. The child should be prepared for a potentially
painful procedure but avoid suggesting pain. The nurse should allow the child his own sensory perception
and evaluation of the procedure.

(B) The nurse should avoid absolute descriptive statements and allow the child his own perception of the
procedure.
(C) The nurse should avoid evaluative statements or descriptions and give the child control in describing
his reactions.
(D) False statements regarding a painful procedure will cause a loss of trust between the child and the
nurse.
326. A 38-year-old female client with a history of chronic schizophrenia, paranoid type, is currently an
outpatient at the local mental health and mental retardation clinic. The client comes in once a week for
medication evaluation and/or refills. She self-administers haloperidol 5 mg twice a day and benztropine 1
mg once a day. During a recent clinic visit, she says to the nurse, "I can't stay still at night. I toss and turn
and can't fall asleep." The nurse suspects that she may be experiencing:
A. Akathisia
B. Akinesia
C. Dystonia
D. Opisthotonos
Answer: A
Explanation:
(A) Akathisia, or motor restlessness, is a reversible EPS frequently associated with the administration of
antipsychotic drugs such as haloperidol.
(B) Akinesia, or muscular or motor retardation, is an example of reversible EPS frequently associated with
the administration of major tranquilizers such as haloperidol.
(C) Acute dystonic reactions, bizarre and severe muscle contractions usually of the tongue, face, neck or
extraocular muscles, are examples of EPS.
(D) Opisthotonos, a severe type of whole-body dystonic reaction in which the head and heels are bent
backward while the body is bowed forward, is an example of EPS.
327. A client is receiving peritoneal dialysis. He has been taught to warm the dialyzing fluid prior to
instilling it because:
A. Warmed solution helps keep the body temperature maintained within a normal range during instillation
B. Warmed solution helps dilate the peritoneal blood vessels
C. Warmed solution decreases the risk of peritoneal infection
D. Warmed solution promotes a relaxed abdominal muscle

Answer: B
Explanation:
(A) Instilling a cool solution does not significantly lower the body temperature during peritoneal dialysis.
(B) Warmed solution does help dilate the peritoneal blood vessels, facilitating the exchange of fluids.
(C) Warming the dialysate does not decrease the risk of peritoneal infection. Sterile technique decreases
this risk.
(D) Relaxing the abdominal muscles does not facilitate peritoneal dialysis.
328. The nurse would expect to include which of the following when planning the management of the
client with Lyme disease?
A. Complete bed rest for 6-8 weeks
B. Tetracycline treatment
C. IV amphotericin B
D. High-protein diet with limited fluids
Answer: B
Explanation:
(A) The client is not placed on complete bed rest for 6 weeks.
(B) Tetracycline is the treatment of choice for children with Lyme disease who are over the age of 9.
(C) IV amphotericin B is the treatment for histoplasmosis.
(D) The client is not restricted to a high-protein diet with limited fluids.
329. A client is 2 hours post ventriculoperitoneal shunt placement. How should the nurse position the
client?
A. Head of bed elevated 30 degrees on nonoperative side
B. Head of bed elevated 30 degrees on operative side
C. Bed flat on operative side
D. Bed flat on nonoperative side
Answer: D
Explanation:
(A) Elevation of head on nonoperative side would be the position for the late postoperative period.
(B) Positioning on operative side puts pressure on the suture lines and on the shunt valve. Elevation of
head in immediate postoperative period may cause rapid reduction of cerebrospinal fluid.
(C) Placement on operative side puts pressure on the suture lines and shunt valve.

(D) Flat position on nonoperative side in the immediate postoperative period prevents pressure on shunt
valve and rapid reduction in cerebrospinal fluid.
330. A male client is experiencing auditory hallucinations. His nurse enters the room and he tells her that
his mother is talking to him, and he will take his medicine after she leaves. The nurse looks around the
room and sees that she and the client are the only ones in the room. The nurse's most therapeutic response
will be:
A. "I don't see your mother in the room. Let's talk about how you're feeling."
B. "OK, I'll come back later when you're feeling more like taking your medicine."
C. "She may be here, but I can't see her."
D. "Why don't you finish talking to her, and I'll wait."
Answer: A
Explanation:
(A) This response uses the principle of reality orientation by the nurse telling the client that he or she does
not see anything, but it does recognize his feelings.
(B) This response does not make it clear that the nurse does not see anyone else in the room, and the nurse
leaves the client alone to continue hallucinating.
(C) This response leaves room for doubt; the nurse is further confusing the client by this statement.
(D) This response reinforces the hallucination and implies that the nurse sees his mother, too.
331. A client who is 7 months pregnant is diagnosed with pyelonephritis. The nurse anticipates the
physician ordering:
A. Oxytocin
B. Magnesium sulfate (MgSO4)
C. Ampicillin
D. Tetracycline
Answer: C
Explanation:
(A) Oxytocin is prescribed to stimulate uterine contractions.
(B) MgSO4 is a central nervous system depressant prescribed to prevent and control convulsions related to
preeclampsia.
(C) Ampicillin is a penicillin derivative with no known teratogenic effects. This is the safest antibiotic
during pregnancy.

(D) Tetracycline stains teeth yellow and is not as safe as ampicillin during pregnancy.
332. A baby who was diagnosed with pyloric stenosis has continued to have projectile vomiting. With
prolonged vomiting, the infant is prone to:
A. Respiratory acidosis
B. Respiratory alkalosis
C. Metabolic acidosis
D. Metabolic alkalosis
Answer: D
Explanation:
(A) Respiratory acidosis is the result of problematic ventilation. Plasma pH decreases, while plasma PCO 2
and plasma HCO3 increase.
(B) Respiratory alkalosis results from increased respiratory rate and depth. Plasma pH increases, while
plasma PCO2 and plasma HCO3 decrease.
(C) Metabolic acidosis occurs when there is strong acid gain in the body. Plasma pH, PCO 2, and HCO3
decrease.
(D) Increased risk for metabolic alkalosis is due to a loss of hydrogen ions; depletion of potassium,
sodium, and chloride when vomiting occurs. Plasma pH and plasma PCO2 increase; plasma HCO3 may
decrease and then increase to compensate.
333. The nurse who is caring for a client with pneumonia assesses that the client has become increasingly
irritable and restless. The nurse realizes that this is a result of:
A. Prolonged bed rest
B. The client's maintaining a semi-Fowler position
C. Cerebral hypoxia
D. IV fluids of 2.5-3 liters in 24 hours
Answer: C
Explanation:
(A) Maintaining bed rest helps to decrease the O2 needs of the tissues, which decreases dyspnea and
workload on the respiratory system.
(B) The semi-Fowler or high-Fowler position is necessary to aid in lessening pressure on the diaphragm
from the abdominal organs, which facilitates comfort and easier breathing patterns.

(C) Cerebral hypoxia causes the client with pneumonia to be increasingly irritable and restless and results
from the client not obtaining enough O2 to meet metabolic needs.
(D) Proper hydration facilitates liquefaction of mucus trapped in the bronchioles and alveoli and enhances
expectoration. Unless contraindicated, a reasonable amount of IV fluids to be administered is at least 2.5-3
liters in a 24-hour period.
334. A client delivered a term infant 1 hour ago. Her uterus on assessment is boggy and is U +1 in contrast
to the previous assessment of U _2. The immediate nursing response is to:
A. Administer methergine IM
B. Remove the retained placental fragments
C. Assist the client to the bathroom and provide cues to stimulate urination
D. Massage the fundus until firm
Answer: D
Explanation:
(A) Methergine is given following placental delivery to promote uterine contractions and prevent
haemorrhage. Methergine may be administered in this clinical situation, but fundal massage would be the
first response.
(B) Removal of retained placental fragments is done by the physician and is not the first response.
(C) If the fundus rises and is deviated, particularly to the right, the nurse should suspect bladder distention
secondary to bladder and urethral trauma associated with birth and decreased bladder tone following
delivery. Therefore, women have a diminished sensation to void.
(D) A boggy fundus rises and is indicative of blood pooling, predisposing the woman to clot formation.
Massage the uterus until firm. Too vigorous massage will result in atonia. Clots may be expelled by a
kneading motion of the uterus by the nurse.
335. The nurse documents a client's surgical incision as having red granulated tissue. This indicates that
the wound is:
A. Infected
B. Not healing
C. Necrotic
D. Healing
Answer: D
Explanation:

(A) The wound is not infected. An infected wound would contain pus, debris, and exudate.
(B) The wound is healing properly.
(C) A necrotic wound would appear black or brown.
(D) The wound is healing properly and is filled with red granulated tissue and fragile capillaries.
336. A 42-year-old client on an inpatient psychiatric unit comments that he was brought to the hospital by
his wife because he had taken too many pills and states, "I just couldn't take it anymore." The nurse's best
response to this disclosure would be:
A. "You shouldn't do things like that, just tell someone you feel bad."
B. "Tell me more about what you couldn't take anymore."
C. "I'm sure you probably didn't mean to kill yourself."
D. "How long have you been in the hospital."
Answer: B
Explanation:
(A) Disapproving gives the impression that the nurse has a right to pass judgment on the client's thoughts,
actions, or ideas.
(B) Giving a broad opening gives the client encouragement to continue with verbalization.
(C) Failing to acknowledge the client's feelings conveys a lack of understanding and empathy.
(D) Changing the subject takes the conversation away from the client and is indicative of the nurse's
anxiety or insensitivity.
337. A client suffering from schizophrenia has been taking chlorpromazine (Thorazine) for 6 months. On
one of his follow-up visits to the mental health center, the nurse reports to the physician that he has
developed tardive dyskinesia. Which of the following symptoms might she have observed in the client to
support this conclusion?
A. High fever, tachycardia, stupor, renal failure
B. Lip smacking, chewing, blinking, lateral jaw movements
C. Photosensitivity, orthostatic hypotension, dry mouth
D. Constipation, blurred vision, drowsiness
Answer: B
Explanation:
(A) These symptoms are found in clients with neuroleptic malignant syndrome.
(B) These symptoms are found in clients with tardive dyskinesia.

(C) These are normal side effects found in clients taking antipsychotic medications.
(D) These are also normal side effects found in clients taking antipsychotic medications.
338. A client has developed congestive heart failure secondary to his myocardial infarction. Discharge diet
instructions should emphasize the reduction or avoidance of:
A. Fresh vegetables and fruit
B. Canned vegetables and fruit
C. Breads, cereals, and rice
D. Fish
Answer: B
Explanation:
(A) Fresh vegetables and fruits are excellent sources of essential vitamins.
(B) Canned and frozen foods have a high sodium content. Labels of all canned foods should be read to
determine if sodium is used in any form.
(C) Bread, cereal, and rice are excellent sources of carbohydrates.
(D) Fish is an excellent source of protein.
339. Which of the following nursing actions is essential to prevent drug-resistant tuberculosis?
A. Monitor liver function.
B. Monitor renal function.
C. Assess knowledge of respiratory isolation.
D. Monitor compliance with drug therapy.
Answer: D
Explanation:
(A) Monitoring liver function will not prevent the development of drug-resistant organisms.
(B) Monitoring renal function will not prevent the development of drug-resistant organisms.
(C) Knowledge of respiratory isolation will reduce transmission of tuberculosis but will not prevent
development of drug-resistant organisms.
(D) Noncompliance with prescribed antituberculosis drug regimen is the primary cause of drug-resistant
organisms. Noncompliance permits the mutation of organisms.
340. A client is a victim of domestic violence. She is now receiving assistance at a shelter for battered
women. She tells the nurse about the cycle of violence that she has been experiencing in her relationship

with her husband of 5 years. In the "tension-building phase," the nurse might expect the client to describe
which of the following?
A. Promises of gifts that her husband made to her
B. Acute battering of the client, characterized by his volatile discharge of tension
C. Minor battering incidents, such as the throwing of food or dishes at her
D. A period of tenderness between the couple
Answer: C
Explanation:
(A) This description is characteristic of the "honeymoon" or "respite" phase.
(B) This description is characteristic of the "battering" phase.
(C) This description is characteristic of the "tension- building" phase prior to the volatile discharge of
tension found in the battering phase.
(D) This description is characteristic of the "honeymoon" or "respite" phase.
341. During the admitting mental health assessment, a client demonstrates involuntary muscular activity.
He has a marked facial tic around the mouth that is distracting to the nurse during the interview. The nurse
recognizes the behavior and documents it as:
A. Dyskinesia
B. Akathisia
C. Echopraxia
D. Echolalia
Answer: A
Explanation:
(A) The client is demonstrating dyskinesia, which is involuntary muscular activity, such as tic, spasm, or
myoclonus.
(B) Akathisia is regular rhythmic movements usually of the lower limbs, such as constant motor
restlessness.
(C) Echopraxia is mimicking the movements of another person.
(D) Echolalia is mimicking the speech of another person.
342. The mother of a client is apprehensive about taking home her 2 year old who was diagnosed with
asthma after being admitted to the emergency room with difficulty breathing and cyanosis. She asks the

nurse what symptoms she should look for so that this problem will not happen again. The nurse instructs
her to watch for the following early symptoms:
A. Fever, runny nose, and hyperactivity
B. Changes in breathing pattern, moodiness, fatigue, and edema of eyes
C. Fatigue, dark circles under the eyes, changes in breathing pattern, glassy eyes, and moodiness
D. Fever, cough, paleness, and wheezing
Answer: C
Explanation:
(A) The child with asthma may not have fever unless there is an underlying infection.
(B) Edema of the eyes will not be present because the child with asthma is more likely to have dehydration
related to excessive water loss during the work of breathing.
(C) All of these symptoms indicate decreased oxygenation and are early symptoms of asthma.
(D) Coughing and wheezing are not early signs of difficulty.
343. A post-lung surgery client is placed on a chest tube drainage system. When explaining to the family
how the system works, the nurse states that the water-seal bottle of a three-bottle chest drainage system
serves which of the following purposes?
A. Collection bottle for drainage
B. Pressure regulator
C. Preventing accumulation of blood around the heart
D. Preventing air from entering the chest upon inspiration
Answer: D
Explanation:
(A) There is a separate collection bottle for drainage as part of a chest drainage system.
(B) In a three-bottle chest drainage system, one bottle serves only as a pressure regulator.
(C) Mediastinal chest tubes prevent accumulation of blood around the heart immediately following heart
surgery.
(D) The purpose of the water-seal bottle in any chest drainage setup is to allow air out of the chest, but not
back in. This negative pressure promotes lung expansion.
344. A client delivered a stillborn male at term. An appropriate action of the nurse would be to:
A. State, "You have an angel in heaven."
B. Discourage the parents from seeing the baby.

C. Provide an opportunity for the parents to see and hold the baby for an undetermined amount of time.
D. Reassure the parents that they can have other children.
Answer: C
Explanation:
(A) This is not a supportive statement. There are also no data to indicate the family's religious beliefs.
(B) Seeing their baby assists the parents in the grieving process. This gives them the opportunity to say
"good-bye."
(C) Parents need time to get to know their baby.
(D) This is not a comforting statement when a baby has died. There are also no guarantees that the couple
will be able to have another child.
345. On the first post partal day, a client tells the nurse that she has been changing her perineal pads every
1/2 hour because they are saturated with bright red vaginal drainage. When palpating the uterus, the nurse
assesses that it is somewhat soft, 1 fingerbreadth above the umbilicus, and midline.
The nursing action to be taken is to:
A. Gently massage the uterus until firm, express any clots, and note the amount and character of lochia
B. Catheterize the client and reassess the uterus
C. Begin IV fluids and administer oxytocic medication
D. Administer analgesics as ordered to relieve discomfort
Answer: A
Explanation:
(A) Gentle massage and expression of clots will let the fundus return to a state of firmness, allowing the
uterus to function as the "living ligature."
(B) A distended bladder may promote uterine atony; however, after determining the bladder is distended,
the nurse would have the client void.
Catheterization is only done if normal bladder function has not returned.
(C) Oxytocic medications are ordered and administered if the uterus does not remain contracted after
gentle massage and determining if the bladder is empty.
(D) The client is not complaining of discomfort or pain; therefore, analgesics are not necessary.
346. A 5-year-old child cries continually in her bed. Her parents have been unsuccessful in assisting her in
expressing her feelings. Which activity should the nurse provide the child to assist her in expressing her
feelings?

A. Books with colorful pictures
B. Music
C. Riding toys
D. Puppets
Answer: D
Explanation:
(A) Books increase cognition, assist with fine motor skills, and augment language development.
(B) Music provides auditory stimulation and large-muscle activity.
(C) Riding toys provide large-muscle activity.
(D) Puppets allow expression of feelings and fears that otherwise could not be directly communicated.
347. A client had a vaginal delivery 3 days ago and is discharged from the hospital on the 2nd day
postpartum. She told the RN, "I need to start exercising so that I can get back into shape. Could you
suggest an exercise I could begin with?'' The RN could suggest which one of the following?
A. Push-ups
B. Jumping jacks
C. Leg lifts
D. Kegel exercises
Answer: D
Explanation:
(A, B, C) This exercise is too strenuous at this time.
(D) This exercise is recommended for the first few days after delivery. It helps to stimulate muscle tonus in
the area of the perineum and the area around the urinary meatus and vagina.
348. A female client is started on warfarin (Coumadin) 5 mg po bid. To adequately evaluate the
effectiveness of the warfarin therapy, the nurse must know that this medication:
A. Dissolves any clots already formed in the arteries
B. Prevents the conversion of prothrombin to thrombin
C. Interferes with the synthesis of vitamin K-dependent clotting factors
D. Stimulates the manufacturing of platelets
Answer: C
Explanation:

(A) Thrombolytic agents (e.g., streptokinase) directly activate plasminogen, dissolving fibrin deposits,
which in turn dissolves clots that have already formed.
(B) Heparin prevents the formation of clots by potentiating the effects of antithrombin III and the
conversion of prothrombin to thrombin.
(C) Warfarin prevents the formation of clots by interfering with the hepatic synthesis of the vitamin K
dependent clotting factors.
(D) Platelets initiate the coagulation of blood by adhering to each other and the site of injury to form
platelet plugs.
349. An 8-year-old child comes to the physician's office complaining of swelling and pain in the knees. His
mother says, "The swelling occurred for no reason, and it keeps getting worse." The initial diagnosis is
Lyme disease. When talking to the mother and child, questions related to which of the following would be
important to include in the initial history?
A. A decreased urinary output and flank pain
B. A fever of over 103F occurring over the last 2-3 weeks
C. Rashes covering the palms of the hands and the soles of the feet
D. Headaches, malaise, or sore throat
Answer: D
Explanation:
(A) Urinary tract symptoms are not commonly associated with Lyme disease.
(B) A fever of 103F is not characteristic of Lyme disease.
(C) The rash that is associated with Lyme disease does not appear on the palms of the hands and the soles
of the feet.
(D) Classic symptoms of Lyme disease include headache, malaise, fatigue, anorexia, stiff neck, generalized
lymphadenopathy, splenomegaly, conjunctivitis, sore throat, abdominal pain, and cough.
350. A husband and wife and their two children, age 9 and age 5, are requesting family therapy. Which of
the following strategies is most therapeutic for the nurse to use during the initial interaction with a family?
A. Always allow the most vocal person to state the problem first.
B. Encourage the mother to speak for the children.
C. Interpret immediately what seems to be going on within the family.
D. Allow family members to assume the seats as they choose.
Answer: D

Explanation:
(A) One will always hear what the most vocal person has to say. It is best to start with the quietest family
member to encourage that person to express emotions.
(B) All family members are encouraged to speak for themselves.
(C) In the initial family assessment, only data collection occurs; interpretations are made later.
(D) Allowing family members to choose their own seats will assist the nurse in assessing the family system
and in determining who feels closer to whom.
351. A client has been diagnosed with thrombophlebitis. She asks, "What is the most likely cause of
thrombophlebitis during my pregnancy?" The nurse explains:
A. Increased levels of the coagulation factors and a decrease in fibrinolysis
B. An inadequate production of platelets
C. An inadequate intake of folic acid during pregnancy
D. An increase in fibrinolysis and a decrease in coagulation factors
Answer: A
Explanation:
(A) During pregnancy, the potential for thromboses increases owing to the increased levels of coagulation
factors and a decrease in the breakdown of fibrin.
(B) An inadequate production of platelets would result in thrombocytopenia with resulting signs and
symptoms of bleeding such as petechiae, haematuria, or hematemesis.
(C) A deficiency of folic acid during pregnancy produces a megaloblastic anaemia. It is usually found in
combination with iron deficiency.
(D) This combination would result in bleeding disorders because more fibrin would be broken down and
fewer clotting factors would be available.
352. Which of the following would differentiate acute from chronic respiratory acidosis in the assessment
of the trauma client?
A. Increased PaCO2
B. Decreased PaO2
C. Increased HCO3
D. Decreased base excess
Answer: C
Explanation:

(A) Increased CO2 will occur in both acute and chronic respiratory acidosis.
(B) Hypoxia does not determine acid-base status.
(C) Elevation of HCO3 is a compensatory mechanism in acidosis that occurs almost immediately, but it
takes hours to show any effect and days to reach maximum compensation. Renal disease and diuretic
therapy may impair the ability of the kidneys to compensate.
(D) Base excess is a non respiratory contributor to acid-base balance. It would increase to compensate for
acidosis.
353. A client is admitted to the hospital with a diagnosis of aplastic anaemia and placed on isolation. The
nurse notices a family member entering the room without applying the appropriate apparel. The nurse will
approach the family member using the following information as a basis for discussion:
A. The risks of exposure of the visitor to infectious organisms is great.
B. Hospital regulations mandate that everyone in the facility adhere to appropriate codes.
C. The client is at extreme risk of acquiring infections.
D. Adherence to the guidelines are the latest Centers for Disease Control and Prevention recommendations
on use of protective apparel.
Answer: C
Explanation:
(A) Although clients with a compromised immune system may acquire infections, the primary emphasis is
on protecting the client.
(B, D) Most people are aware of the guidelines once they see posted signs, so quoting regulations is not
likely to result in consistent adherence to regulations.
(C) Clients with aplastic anaemia have white cell counts of 2000 or lower, making them more vulnerable
to infections from others.
354. A 10-month-old infant's mother says that he takes fresh whole milk eagerly, but that when she offered
him baby foods at 6 months of age, he pushed them out of his mouth. Because he has gained weight
appropriately, she has quit trying to get him to eat other foods. The nurse's response is based on the
knowledge that:
A. Milk intake should be limited to no more than four 8-oz bottles per day and should be followed by ironenriched cereal or other solid foods or juices
B. Milk is an excellent food and will meet his nutritional needs adequately until he is ready to eat solid
foods

C. It is acceptable to continue to give him whole milk and to delay giving solid foods as long as he takes a
vitamin supplement daily
D. He should be started on iron-enriched cereal, meat, vegetables, fruits, and juices prior to bottle feeds.
Milk intake should be limited to 1 qt/day
Answer: D
Explanation:
(A) If the infant is given the bottle first, he will be less likely to be hungry enough to eat the solid foods.
(B) Milk is deficient in iron, vitamin C, zinc, and fluoride. It does not provide an adequate diet.
(C) The vitamin supplement will help, but the infant needs an iron supplement.
(D) Giving the solid food when the infant is hungriest will increase the likelihood that he will eat. The
more solid food he takes, the less milk he will desire.
355. A male client is admitted to the psychiatric unit after experiencing severe depression. He states that he
intends to kill himself, but he asks the nurse not to repeat his intentions to other staff members. Which
response demonstrates understanding and appropriate action on the part of the nurse?
A. "I understand you're depressed, but killing yourself is not a reasonable option."
B. "We need to discuss this further, but right now let's complete these forms."
C. "Don't do that, you have so much to live for. You have a wonderful wife and children. The client in the
next room has no one."
D. "This is very serious. I do not want any harm to come to you. I will have to report this to the rest of the
staff."
Answer: D
Explanation:
(A) To the client, suicide may be a reasonable action and the only one he can cope with at this time.
(B) This response indicates to the client that his intention to commit suicide is not important to the nurse at
this time.
(C) The client is so depressed that he is not able to see the positive aspects of his life. At no time should the
nurse discuss another client's problems in conversation.
(D) This statement tells the client that the nurse recognizes his problem is of a serious nature and will take
all steps necessary to help him.

356. Pin care is a part of the care plan for a client who is in skeletal traction. When assessing the site of pin
insertion, which one of the following findings would the nurse know as an indicator of normal wound
healing?
A. Exudate
B. Crust
C. Edema
D. Erythema
Answer: B
Explanation:
(A) Exudate (moist, active drainage) is a clinical sign of wound infection.
(B) Crust (dry, scaly) is part of the normal stages of wound healing and should not be removed from
around the pin site. It usually sloughs off after the underlying tissue has healed.
(C) Edema (swelling) is a clinical sign of wound infection.
(D) Erythema (redness) is a clinical sign of wound infection.
357. Four days after admission for cirrhosis of the liver, the nurse observes the following when assessing a
male client: increased irritability, asterixis, and changes in his speech pattern. Which of the following
foods would be appropriate for his bedtime snack?
A. Fresh fruit
B. A milkshake
C. Saltine crackers and peanut butter
D. A ham and cheese sandwich
Answer: B
Explanation:
(A) Albumin, a blood volume expander, increases the circulating blood volume by exerting an osmotic pull
on tissue fluids, pulling them into the vascular system. This fluid shift causes an increase in the heart rate
and blood pressure.
(B) Albumin, a blood volume expander, exerts an osmotic pull on fluids in the interstitial spaces, pulling
the fluid back into the circulatory system. This fluid shift causes an increase in the urinary output.
(C) Adventitious breath sounds and dyspnea can occur due to circulatory overload if the albumin is infused
too rapidly.
(D) Chills, fever, itching, and rashes are signs of a hypersensitivity reaction to albumin.

358. After a 10-year-old child with insulin-dependent diabetes mellitus receives her dinner tray, she tells
the nurse that she hates broccoli and wants some corn on the cob. The nurse's appropriate response is:
A. "No vegetable exchanges are allowed."
B. "Corn and other starchy vegetables are considered to be bread exchanges."
C. "Yes, you may exchange any vegetable for any other vegetable."
D. "Yes, but only one-half ear is allowed."
Answer: B
Explanation:
(A) Sites for injection need to be rotated, including abdominal sites, to enhance insulin absorption.
(B) The pinch technique is the most effective method for obtaining skin tightness to allow easy entrance of
the needle to subcutaneous tissues.
(C) Massaging the site of injection facilitates absorption of the insulin.
(D) Changing the needle will break the sterility of the system. It has become acceptable practice to reuse
disposable needles and syringes for 3-7 days.
359. Before completing a nursing diagnosis, the nurse must first:
A. Write goals and objectives
B. Perform an assessment
C. Plan interventions
D. Perform evaluation
Answer: B
Explanation:
(A) Goals and objectives are based on a nursing assessment and diagnosis.
(B) Assessment is the first step of nursing process.
(C) Interventions are nursing actions to meet goals and objectives.
(D) Evaluation process follows nursing interventions.
360. A newborn is admitted to the newborn nursery with tremors, apnea periods, and poor sucking reflex.
The nurse should suspect:
A. Central nervous system damage
B. Hypoglycemia
C. Hyperglycemia
D. These are normal newborn responses to extrauterine life

Answer: B
Explanation:
(A) Central nervous system damage presents as seizures, decreased arousal, and absence of newborn
reflexes.
(B) In a diabetic mother, the infant is exposed to high serum glucose. The fetal pancreas produces large
amounts of insulin, which causes hypoglycemia after birth.
(C) Hypoglycemia is a common newborn problem. Increased insulin production causes hypoglycemia, not
hyperglycemia.
(D) These are not normal adaptive behaviors to extrauterine life.
361. A client is pregnant with her second child. Her last menstrual period began on January 15. Her
expected date of delivery would be:
A. October 8
B. October 15
C. October 22
D. October 29
Answer: C
Explanation:
(A) Incorrect application of Nagele's rule: correctly subtracted 3 months but subtracted 7 days rather than
added.
(B) Incorrect application of Nagele's rule: correctly subtracted 3 months but did not add 7 days.
(C) Correct application of Nagele's rule: correctly subtracted 3 months and added 7 days.
(D) Incorrect application of Nagele's rule: correctly subtracted 3 months but added 14 days instead of 7
days.
362. A client returns to the cardiovascular intensive care unit following his coronary artery bypass graft. In
planning his care, the most important electrolyte the nurse needs to monitor will be:
A. Chloride
B. HCO3
C. Potassium
D. Sodium
Answer: C
Explanation:

(A) Chloride, HCO3, and sodium will need to be monitored, but monitoring these electrolytes is not as
important as potassium monitoring.
(B) Chloride, HCO3, and sodium will need to be monitored, but monitoring these electrolytes is not as
important as potassium monitoring.
(C) Potassium will need to be closely monitored because of its effects on the heart. Hypokalaemia could
result in supraventricular tachyarrhythmias.
(D) Chloride, HCO3, and sodium will need to be monitored, but monitoring these electrolytes is not as
important as potassium monitoring.
363. A client is pleased about being pregnant, yet states, "It is really not the best time, but I guess it will be
OK." The nurse's assessment of this response is:
A. Initial maternal-infant bonding may be poor.
B. Client may have a poor relationship with her husband.
C. This response is normal in the first trimester.
D. This response is abnormal, to be re-evaluated at the next visit.
Answer: C
Explanation:
(A) Ambivalence is normal during the first trimester. Reva Rubin addresses the issue of "not now" in the
first trimester. The statement still leaves room for exploration.
(B) There are no data to support this. This statement by the mother still leaves room for exploration.
(C) Ambivalence is normal during the first trimester. Reva Rubin addresses the issue of "not now." This
fact should be shared with the mother during further exploration of the comment.
(D) It is not abnormal. If it were, another month would also be too long to wait.
364. On admission to the post partal unit, the nurse's assessment identifies the client's fundus to be soft, 2
fingerbreadths above the umbilicus, and deviated to the right. This is most likely an indication of:
A. Normal involution
B. A full bladder
C. An infection pain
D. A haemorrhage
Answer: B
Explanation:
(A) Immediately after expulsion of the placenta, the fundus should be in the midline and remain firm.

(B) A boggy displaced uterus in the immediate postpartum period is a sign of urinary distention. Because
uterine ligaments are stretched, a full bladder can displace the uterus.
(C) Symptoms of infection may include unusual uterine discomfort, temperature elevation, and foulsmelling lochia. The stem of this question does not address any of these factors.
(D) While excessive bleeding is associated with a soft, boggy uterus, the stem of this question includes
displacement of the uterus, which is more commonly associated with bladder distention.
365. A 30-year-old female client is receiving antineoplastic chemotherapy. Which of the following
symptoms should especially concern the nurse when caring for her?
A. Respiratory rate of 16 breaths/min
B. Pulse rate of 80 bpm
C. Complaints of muscle aches
D. A sore throat
Answer: D
Explanation:
(A) A respiratory rate of 16 breaths/min is normal and is not a cause for alarm.
(B) A pulse rate of 80 bpm is normal and is not a cause for alarm.
(C) Complaints of muscle aches are unrelated to her receiving chemotherapy. There may be other causes
related to her hospital stay or the disease process.
(D) A sore throat is an indication of a possible infection. A client receiving chemotherapy is at risk of
neutropenia. An infection in the presence of neutropenia can result in a life-threatening situation.
366. A 35-weeks-pregnant client is undergoing a nonstress test (NST). During the 20-minute examination,
the nurse notes three fetal movements accompanied by accelerations of the fetal heart rate, each 15 bpm,
lasting 15 seconds. The nurse interprets this test to be :
A. Nonreactive
B. Reactive
C. Positive
D. Negative
Answer: B
Explanation:
(A) In a nonreactive NST, the criteria for reactivity are not met.

(B) A reactive NST shows at least two accelerations of FHR with fetal movements, each 15 bpm, lasting
15 seconds or more, over 20 minutes.
(C, D) This term is used to interpret a contraction stress test (CST), or oxytocin challenge test, not an NST.
367. A client has returned to the unit following a left femoral popliteal bypass graft. Six hours later, his
dorsalis pedis pulse cannot be palpated, and his foot is cool and dusky. The nurse should:
A. Continue to monitor the foot
B. Notify the physician immediately
C. Reposition and reassess the foot
D. Assure the client that his foot is fine
Answer: B
Explanation:
(A) The client is losing blood supply to his left foot. Continuing to monitor the foot will not help restore
the blood supply to the foot.
(B) The physician should be notified immediately because the client is losing blood supply to his left foot
and is in danger of losing the foot and/or leg.
(C) The presenting symptoms are of an emergency nature and require immediate intervention.
(D) This action would be giving the client false assurance.
368. An elective saline abortion has been performed on a 3- week primigravida. Following the procedure,
the nurse should be alert for which early side effect?
A. Water satiety
B. Thirst
C. Edema
D. Diabetes insipidus
Answer: B
Explanation:
(A) If the client is experiencing water satiety, there is no more desire for water.
(B) Absorption of saline into circulation rather than into amniotic sac increases serum sodium and desire
for water.
(C) Edema can be a late side effect caused by water intoxication.
(D) Diabetes insipidus occurs as a result of deficient antidiuretic hormone.

369. A 29-year-old client is diagnosed with borderline personality disorder. He has aroused the nurse's
anger by using a condescending tone of voice with other clients and staff persons. Which of the following
statements from the nurse would be most appropriate in acknowledging feelings regarding the client's
behavior?
A. "I feel angry when I hear that tone of voice."
B. "You make me angry when you talk to me that way."
C. "Are you trying to get me angry?"
D. "Why do you treat me that way?"
Answer: A
Explanation:
(A)The nurse appropriately states how he or she feels when the client speaks in a condescending manner.
(B) This statement indicates that the client has control over the nurse. No one makes another person angry;
each individual has a choice.
(C) "Why" questions usually put a person on the defensive. In addition, the client cannot "make" the nurse
angry. The client does not have that control.
(D) Again, a "why" statement places the client on the defensive.
370. A 16-year-old diabetic girl has been selected as a cheerleader at her school. She asks the nurse
whether she should increase her insulin when she is planning to attend cheerleading practice sessions
lasting from 8 to 11 AM. The most appropriate answer would be:
A. "You should ask your doctor about this."
B. "Yes, increase your insulin by 1 U for each hour of practice because exercise causes the body to need
more insulin."
C. "No, do not increase your insulin. Exercise will not affect your insulin needs."
D. "No, do not increase your insulin, but eating a snack prior to practice exercise will make insulin more
effective and move more glucose into the cells."
Answer: D
Explanation:
(A) A nurse can give this information to a client.
(B) Exercise makes insulin more efficient in moving more glucose into the cells. No more insulin is
needed.
(C) Exercise makes insulin more efficient unless the diabetes is poorly controlled.
(D) Exercise makes insulin more efficient in moving more glucose into the cells.

371. A client has a history of alcoholism. He is currently diagnosed with cirrhosis of the liver. The nurse
would expect him to be on which type of diet?
A. High protein and high calorie
B. High calorie and high carbohydrate
C. Low-fat 2-g sodium diet
D. High protein and high fat
Answer: B
Explanation:
(A) A high-protein diet is contraindicated in hepatic disease.
(B) High carbohydrates provide high caloric content to prevent tissue catabolism.
(C) A low-fat 2-g sodium diet is a cardiac diet; however, a low-fat diet would be beneficial.
(D) A high-protein and high-fat diet is contraindicated in hepatic disease.
372. In cleansing the perineal area around the site of catheter insertion, the nurse would:
A. Wipe the catheter toward the urinary meatus
B. Wipe the catheter away from the urinary meatus
C. Apply a small amount of talcum powder after drying the perineal area
D. Gently insert the catheter another 12 inch after cleansing to prevent irritation from the balloon
Answer: B
Explanation:
(A) Wiping toward the urinary meatus would transport microorganisms from the external tubing to the
urethra, thereby increasing the risk of bladder infection.
(B) Wiping away from the urinary meatus would remove microorganisms from the point of insertion of the
catheter, thereby decreasing the risk of bladder infection.
(C) Talcum powder should not be applied following catheter care, because powders contribute to moisture
retention and infection likelihood.
(D) The catheter should never be inserted further into the urethra, because this would serve no useful
purpose and would increase the risk of infection.
373. A pregnant client is having a nonstress test (NST). It is noted that the fetal heart beat rises 20 bpm,
lasting 20 seconds, every time the fetus moves. The nurse explains that:
A. The test is inconclusive and should be repeated

B. Further testing is needed
C. The test is normal and the fetus is reacting appropriately
D. The fetus is distressed
Answer: C
Explanation:
(A) The test results were normal, so there would be no need to repeat to determine results.
(B) There are no data to indicate further tests are needed, because the result of the NST was normal.
(C) An NST is reported as reactive if there are two to three increases in the fetal heart rate of 15 bpm,
lasting at least 15 seconds during a 15-minute period.
(D) The NST results were normal, so there was no fetal distress.
374. When planning care for a 9-year-old client, the nurse uses which of the most effective means of
helping siblings cope with their feelings about a brother who is terminally ill?
A. Open discussion and understanding
B. Play-acting out feelings in different roles
C. Storytelling
D. Drawing pictures
Answer: B
Explanation:
(A) When dealing with grief, siblings are usually most comfortable initially with open discussion.
(B) Assuming different roles allows children to act out their feelings without fear of reprisals and to gain
insight and control.
(C) This method may be helpful, but having the child take an active part through role playing is more
effective.
(D) This technique may be helpful, but being an active participant through role playing is more effective.
375. A 33-year-old client was brought into the emergency room unconscious, and it is determined that
surgery is needed. Informed consent must be obtained from his next of kin.
The sequence in which the next of kin would be asked for the consent would be:
A. Parent, spouse, adult child, sibling
B. Spouse, adult child, parent, sibling
C. Spouse, parent, sibling, adult child
D. Parent, spouse, sibling, adult child

Answer: B
Explanation:
(A) Spouse and adult child would be asked before a parent.
(B) The order of kin relationship for an adult, as determined from legal intestate succession, is usually
spouse, adult child, parent, sibling.
(C) Parent and sibling would be asked after adult child.
(D) Spouse and adult child would be asked before parent. Sibling would be asked last.
376. After 7 hours in restraints and a total of 30-mg haloperidol in divided doses, a client complains of
stiffness in his neck and his tongue "pulling to one side." These extrapyramidal symptoms (EPS) will most
likely be relieved by the administration of:
A. Lorazepam (Ativan)
B. Benztropine (Cogentin)
C. Thiothixene (Navane)
D. Flurazepan (Dalmane)
Answer: B
Explanation:
(A) Lorazepam is an antianxiety agent that produces muscle relaxation and inhibits cortical and limbic
arousal. It has no action in the basal ganglia of the brain.
(B) Benztropine acts to reduce EPS by blocking excess CNS cholinergic activity associated with dopamine
deficiency in the basal ganglia by displacing acetylcholine at the receptor site.
(C) Thiothixene is an antipsychotic known to block dopamine in the limbic system, thereby causing EPS.
(D) Flurazepan is a hypnotic that acts in the limbic system, thalamus, and hypothalamus of the CNS to
produce sleep. It has no known action in the vasal ganglia.
377. A client is being admitted to the labor and delivery unit. She has had previous admissions for "false
labor." Which clinical manifestation would be most indicative of true labor?
A. Increased bloody show
B. Progressive dilatation and effacement of the cervix
C. Uterine contractions
D. Decreased discomfort with ambulation
Answer: B
Explanation:

(A) Bloody show is considered a sign of imminent labor, which usually begins in 24-48 hours. An increase
in bloody show is an indication that the cervix is changing.
(B) Contractions of true labor produce progressive cervical effacement and dilatation.
(C) Contractions of false labor may mimic those of true labor. However, the contractions of false labor do
not produce progressive effacement and dilatation of the cervix.
(D) In true labor, the discomfort is not relieved by ambulation; walking may intensify the discomfort.
378. In healthcare settings, nurses must be familiar with primary, secondary, and tertiary levels of care. As
a nurse in the community, which of the following interventions might be a primary prevention strategy?
A. Crisis intervention with an intoxicated teenager whose mother just committed suicide
B. Referring a client who has been on a detoxification unit to a rehabilitation center
C. Teaching fifth-grade children the harmful effects of substance abuse
D. Counseling a client with post-traumatic stress disorder
Answer: C
Explanation:
(A) The teenager is already coping ineffectively and requires early detection and treatment, which is
secondary prevention.
(B) The client must be sent to a rehabilitation unit, which requires tertiary prevention.
(C) Reducing the incidence of disease through education supports primary prevention.
(D) A client with identified symptoms of post-traumatic stress disorder requires intervention by treatment.
379. A 15-year-old child is admitted to the paediatric unit with a diagnosis of thalassemia. Which of the
following would be included in educating the mother and child as part of discharge planning?
A. Give oral iron medication every day.
B. Have the child's blood pressure monitored every week.
C. Know the signs and symptoms of iron overload.
D. Keep exercise at a minimum to reduce stress.
Answer: C
Explanation:
(A) Oral iron supplements are contraindicated in thalassemia.
(B) Although heart failure may be an end result of this disease, this action is unnecessary.
(C) Iron overload is a potential complication of frequent blood transfusions of children with thalassemia.
(D) Children should be encouraged to pursue activities related to their exercise tolerance.

380. What is the most effective method to identify early breast cancer lumps?
A. Mammograms every 3 years
B. Yearly checkups performed by physician
C. Ultrasounds every 3 years
D. Monthly breast self-examination
Answer: D
Explanation:
(A) Mammograms are less effective than breast self-examination for the diagnosis of abnormalities in
younger women, who have denser breast tissue. They are more effective forwomen older than 40.
(B) Up to 15% of early-stage breast cancers are detected by physical examination; however, 95% are
detected by women doing breast self-examination.
(C) Ultrasound is used primarily to determine the location of cysts and to distinguish cysts from solid
masses.
(D) Monthly breast self-examination has been shown to be the most effective method for early detection of
breast cancer. Approximately 95% of lumps are detected by women themselves.
381. An IDDM client's condition stabilizes. He begins to receive a daily injection of NPH insulin at 6:30
AM. The nurse can most likely expect a hypoglycaemic reaction to occur that same day at :
A. 8:30 AM-10:30 AM
B. 2:30 PM-4:30 PM
C. 7:30 PM-9:30 PM
D. 10:30 PM-11:30 PM
Answer: B
Explanation:
(A) This time describes the time of onset of NPH insulin's action, rather than its peak effect.
(B) NPH insulin, an intermediate acting insulin, usually begins to lower serum glucose levels about 2
hours after administration. The action of NPH insulin peaks 8-14 hours after administration. It has a 20-30
hour duration.
(C) The time stated is not the time of peak action for NPH insulin administered at 6:30 AM.
(D) The time stated is not the time of peak action for NPH insulin administered at 6:30 AM.

382. A 50-year-old depressed client has recently lost his job. He has been reluctant to leave his hospital
room. Nursing care would include:
A. Forcing the client to attend all unit activities
B. Encouraging the client to discuss why he is so sad
C. Monitoring elimination patterns
D. Providing sensory stimulation
Answer: C
Explanation:
(A) The client should be encouraged to attend the unit activities. The nurse and client should choose a few
activities for the client to attend that will be positive experiences for him.
(B) The nurse should encourage the client to discuss his feelings and to begin to deal with the depression.
(C) Depressed persons often have little appetite and poor fluid intake. Constipation is common.
(D) A calm, consistent level of stimuli is most effective. Sensory deprivation and overstimulation should
be avoided.
383. Discharge teaching was effective if the parents of a child with atopic dermatitis could state the
importance of:
A. Maintaining a high-humidified environment
B. Furry, soft stuffed animals for play
C. Showering 3-4 times a day
D. Wrapping hands in soft cotton gloves
Answer: D
Explanation:
(A) Maintaining a low-humidified environment.
(B) Avoiding furry, soft stuffed animals for play, which may increase symptoms of allergy.
(C) Avoiding showering, which irritates the dermatitis, and encouraging bathing 4 times a day in colloid
bath for temporary relief.
(D) Wrapping hands in soft cotton gloves to prevent skin damage during scratching.
384. A client has been admitted to the labor and delivery unit in active labor. After assessing her, the RN
notes that the client's fetus position is left occipital posterior. Which of the following statements best
describes what this means to the labor process:
A. Decreases the overall time of the labor process

B. Prolongs the client's first stage of labor
C. Decreases the time of the client's first stage of labor
D. Prolongs the client's third stage of labor
Answer: B
Explanation:
(A) Posterior position causes a larger diameter of the fetal head to enter the pelvis than an anterior
position. Pressure on the sacral nerves is increased, and it takes the fetus a longer time to enter the pelvic
inlet.
(B) This position will prolong the first stage of labor. When the larger diameter of the fetal head enters the
pelvis first, it will have a more difficult time accommodating to the pelvis; therefore, it will take a longer
time for the fetus to move through the pelvis.
(C) It will increase the time of labor because the larger diameter of the fetal head will have a more difficult
time accommodating to the pelvic inlet and thus will move through the pelvis slower.
(D) In the third stage of labor the placenta is delivered; therefore, the infant has been delivered.
385. On morning rounds, the nurse found a manic-depressive client who is taking lithium in a confused
mental state, vomiting, twitching, and exhibiting a coarse hand tremor. Which one of the following nursing
actions is essential at this time?
A. Administer her next dosage of lithium, and then call the physician.
B. Withhold her lithium, and report her symptoms to the physician.
C. Place her on NPO to decrease the excretion of lithium from her body, and call the physician.
D. Contact the lab and request a lithium level in 30 minutes, and call the physician.
Answer: B
Explanation:
(A) The client has lithium toxicity, and the nurse must withhold further dosages.
(B) Because of her level of toxicity, further lithium could cause coma and death. The nurse needs further
orders from the physician to stabilize the client's lithium level.
(C) Ensuring adequate intake of sodium chloride will promote excretion of lithium and will assist in
managing the client's lithium toxicity.
(D) A lithium blood level must be drawn immediately to determine the seriousness of the toxicity and to
provide the physician with data for medical orders.

386. A female client comes for her second prenatal visit. The nurse-midwife tells her, "Your blood tests
reveal that you do not show immunity to the German measles." Which notation will the nurse include in
her plan of care for the client? "Will need . . .
A. Rh-immune globulin at the next visit"
B. Rh-immune globulin within 3 days of delivery"
C. Rubella vaccine at the next visit"
D. Rubella vaccine after delivery on the day of discharge"
Answer: D
Explanation:
(A) Rh immune globulin is given to Rh-negative mothers to prevent the maternal Rh immune response.
(B) Rh immune globulin is given to Rh-negative mothers to prevent the maternal Rh immune response.
(C) The rubella vaccine is not given during pregnancy because of its teratogenicity.
(D) Nonimmune mothers are vaccinated early in the postpartum period to prevent future infection with the
rubella virus.
387. A 70-year-old female client is admitted to the medical intensive care unit with a diagnosis of
cerebrovascular accident (CVA). She is semi comatose, responding to pain and change in position.
She is unable to speak or cough. In planning her nursing care for the first 24 hours following a CVA, which
nursing diagnosis should receive the highest priority?
A. Ineffective airway clearance related to immobility, ineffective cough, and decreased level of
consciousness
B. Altered cerebral tissue perfusion related to pathophysiological changes that decrease blood flow
C. Potential for injury related to impaired mobility and seizures
D. Impaired verbal communication related to aphasia
Answer: A
Explanation:
(A) An effective airway is necessary to prevent hypoxia and subsequent cardiac arrest.
(B) Cerebral tissue perfusion is necessary to preserve remaining cerebral tissue, but this goal is secondary
to maintenance of an effective airway.
(C) While prevention of injury is important, it is secondary to maintaining an effective airway and cerebral
tissue perfusion.
(D) Impaired verbal communication is not life threatening in the acute phase of recovery. It is the lowest
priority of the nursing diagnoses listed.

388. A 72-year-old male client had the Foley catheter that was inserted during the transurethral resection of
his prostate removed today. He is concerned about the urinary incontinence he is having since removal of
the Foley catheter. The nurse explains that:
A. He should not be concerned about it because it will resolve quickly
B. This is usually temporary
C. The nurse will keep him dry, and he should notify the nurse when this happens
D. This is related to the bladder spasms and will soon stop
Answer: B
Explanation:
(A) This problem is temporary, but it may take some time to resolve, especially in an older man.
(B) This problem is usually temporary, but it may take some time to resolve.
(C) Keeping the client dry will not relieve his anxiety about his incontinence.
(D) The bladder spasms are not the cause of the client's incontinence.
389. When a client with pancreatitis is discharged, the nurse needs to teach him how to prevent another
occurrence of acute pancreatitis. Which of the following statements would indicate he has an
understanding of his disease?
A. "I will not eat any raw or uncooked vegetables."
B. "I will limit my alcohol to one cocktail per day."
C. "I will look into attending Alcoholics Anonymous meetings."
D. "I will report any changes in bowel movements to my doctor."
Answer: C
Explanation:
(A) Raw or uncooked vegetables are all right to eat post discharge.
(B) This client must avoid any alcohol intake.
(C) The client displays awareness of the need to avoid alcohol.
(D) This action would be pertinent only if fatty stools associated with chronic hepatitis were the problem.
390. A pregnant client continues to visit the clinic regularly during her pregnancy. During one of her visits
while lying supine on the examining table, she tells the RN that she is becoming lightheaded. The RN
notices that the client has pallor in her face and is perspiring profusely. The first intervention the RN
should initiate is to:

A. Place the examining table in the Trendelenburg position
B. Assess the client to see if she is having vaginal bleeding
C. Obtain the client's vital signs immediately
D. Help the client to a sitting position
Answer: D
Explanation:
(A) This position would cause the gravid uterus to bear the increased pressure of the vena cava, which
could lead to maternal hypotension, in turn causing the client to continue to have pallor and to feel lightheaded.
(B) This would not be the first intervention the RN should initiate. The RN should understand the supine
position and its effect on the gravid uterus and vena cava.
(C) The RN's first intervention should be one that helps to alleviate the client's symptoms. Obtaining her
vital signs will not alleviate her symptoms.
(D) This would move the gravid uterus off of the client's vena cava, which would alleviate the maternal
hypotension that is the cause of her symptoms.
391. A client is having a pneumonectomy done today, and the nurse is planning her postoperative care.
Nursing interventions for a postoperative left pneumonectomy would include:
A. Monitoring the chest tubes
B. Positioning the client on the right side
C. Positioning the client in semi-Fowler position with a pillow under the shoulder and back
D. Monitoring the right lung for an increase in rales
Answer: D
Explanation:
(A) Chest tubes are usually not necessary in a pneumonectomy because there is no lung to re-expand on
the operative side.
(B) The pneumonectomy client should be positioned on the back or operated side because the sutured
bronchial stump may open, allowing fluid to drain into the unoperated side and drown the client.
(C) The client should not have a pillow under the shoulder and back because of the subscapular incision.
(D) Rales are commonly heard over the base of the remaining lung, but an increase could indicate
circulatory overload and therefore should be closely monitored.

392. Children often experience visual impairments. Refractive errors affect the child's visual activity. The
main refractive error seen in children is myopia. The nurse explains to the child's parents that myopia may
also be described as:
A. Cataracts
B. Farsightedness
C. Near sightedness
D. Lazy eye
Answer: C
Explanation:
(A) Cataracts are not considered refractive errors. Cataracts can be described as opacity of the lens.
(B) Hyperopiais the term for farsightedness. One can see objects at a distance more clearly than close
objects.
(C) Myopiais the term for near sightedness. Objects that are close in distance are more clearly seen.
(D) Lazyeye refers to strabismus or misalignment of the eyes.
393. One of the most dramatic and serious complications associated with bacterial meningitis is
Waterhouse- Friderichsen syndrome, which is:
A. Peripheral circulatory collapse
B. Syndrome of inappropriate antiduretic hormone
C. Cerebral edema resulting in hydrocephalus
D. Auditory nerve damage resulting in permanent hearing loss
Answer: A
Explanation:
(A) Waterhouse-Friderichsen syndrome is peripheral circulatory collapse, which may result in extensive
and diffuse intravascular coagulation and thrombocytopenia resulting in death.
(B) Syndrome of inappropriate antidiuretic hormone is a complication of meningitis, but it is not
Waterhouse-Friderichsen syndrome.
(C) Cerebral edema resulting in hydrocephalus is a complication of meningitis, but it is not WaterhouseFriderichsen syndrome.
(D) Auditory nerve damage resulting in permanent hearing loss is a complication of meningitis, but it is
not Waterhouse- Friderichsen syndrome.

394. When inspecting a cardiovascular client, the nurse notes that he needs to sit upright to breathe. This
behavior is most indicative of:
A. Pericarditis
B. Anxiety
C. Congestive heart failure
D. Angina
Answer: C
Explanation:
(A) Pericarditis can cause dyspnea but primarily causes chest pain.
(B) Anxiety can cause dyspnea resulting in SOB, yet it is not typically influenced by degree of head
elevation.
(C) The inability to oxygenate well without being upright is most indicative of congestive heart failure,
due to alveolar drowning.
(D) Angina causes primarily chest pain; any SOB associated with angina is not influenced by body
position.
395. Several months after antibiotic therapy, a child is readmitted to the hospital with an exacerbation of
osteomyelitis, which is now in the chronic stage. The mother appears anxious and asks what she could
have done to prevent the exacerbation. The nurse's response is based on the knowledge that chronic
osteomyelitis:
A. Is caused by poor physical conditions or poor nutrition
B. Often results from unhygienic conditions or an unclean environment
C. Is directly related to sluggish circulation in the affected limb
D. May develop from sinuses in the involved bone that retain infectious material
Answer: D
Explanation:
(A) Poor nutrition and/or poor physical conditions are factors that predispose to the development of
osteomyelitis but do not cause it.
(B) An unclean or unhygienic environment may predispose to the development of chronic osteomyelitis,
but it does not cause an exacerbation of the previous infection.
(C) Sluggish circulation through the medullary cavity during acute osteomyelitis may delay healing, but it
does not cause the disease to become chronic.

(D) Areas of sequestrum may be surrounded by dense bone, become honeycombed with sinuses, and retain
infectious organisms for a long time.
396. Blood work reveals the following lab values for a client who has been diagnosed with anorexia
nervosa: haemoglobin 9.6 g/dL, hemocrit 27%, potassium 2.7 mEq/L, sodium 126 mEq/L. The greatest
danger to her at this time is:
A. Hypoglycemia from low-carbohydrate intake
B. Possible cardiac dysrhythmias secondary to hypokalemia
C. Dehydration from vomiting
D. Anoxia secondary to anemia
Answer: B
Explanation:
(A) There is no lab data to support hypoglycemia.
(B) Hypokalemia, caused by vomiting and decreased dietary intake of potassium, can result in lifethreatening dysrhythmias.
(C) Evidence of dehydration is not life threatening at this time, although fluid volume deficit does need to
be addressed.
(D) The client's haemoglobin does not reflect a life threatening value sufficient to render the client anoxic.
397. A client is pregnant for the fourth time and has had three normal vaginal deliveries. She is in active
labor and fully dilated. Suddenly she calls, "Nurse, the baby is coming." As the nurse responds to her call,
which one of the following observations should the nurse make first?
A. Inspect the perineum.
B. Time the contractions.
C. Prepare a sterile area for delivery.
D. Auscultate for fetal heart rate (FHR).
Answer: A
Explanation:
(A) The nurse must assess the labor status to determine if birth is imminent. The nurse may note perineal
bulging, crowning, or birth of the head to ascertain labor status.
(B) Assessing uterine contractions is one intervention to ascertain labor status. Based on the client's cry, it
is not the intervention of choice.

(C) If delivery of the infant is imminent, preparing a clean or sterile area for delivery is appropriate, but
labor status must be established, whether delivery is imminent, by perineal assessment.
(D) Assessing FHR is one intervention to ascertain fetal well-being. Based on the client's cry, this is not the
intervention of choice.
398. In working with a manipulative client, which of the following nursing interventions would be most
appropriate?
A. Bargaining with the client as a strategy to control the behavior
B. Redirecting the client
C. Providing a consistent set of guidelines and rules
D. Assigning the client to different staff persons each day
Answer: C
Explanation:
(A) This answer is incorrect. Bargaining is a manipulative act, which the nurse could expect from the
client.
(B) This answer is incorrect. Confrontation is an effective nursing strategy with manipulative behavior.
Redirection is appropriate for the client who is out of touch with reality.
(C) This answer is correct. Manipulative clients must abide by consistent rules.
(D) This answer is incorrect.
Manipulation is kept at a minimum if the same staff person is assigned to the client. Often the client will
attempt to play staff persons against each other.
399. The nurse is trying to help a mother understand what is happening with her son who has recently been
diagnosed with paranoid schizophrenia. At present, he is experiencing hallucinations and delusions of
persecution and suffers from poor hygiene. The nurse can best help her understand her son's condition by
which of the following statements?
A. "Sometimes these symptoms are caused by an overstimulation of a chemical called dopamine in the
brain."
B. "Has anyone in your family ever had schizophrenia?"
C. "If your son has a twin, he probably will eventually develop schizophrenia, too."
D. "Some of his symptoms may be a result of his lack of a strong mother-child bonding relationship."
Answer: A
Explanation:

(A) The most plausible theory to date is that dopamine causes an overstimulation in the brain, which
results in the psychotic symptoms.
(B) This statement will only create anxiety in the mother, and the genetic theory is only one of the
etiological factors.
(C) This statement will cause the mother much alarm, and nothing was mentioned about any other child.
(D) The mother child relationship is one of the previous theories examined, but it is not one to be
emphasized, thereby causing a lot of anxiety for the mother.
400. The usual treatment for diabetes insipidus is with IM or SC injection of vasopressin tannate in oil.
Nursing care related to the client receiving IM vasopressin tannate would include:
A. Weigh once a week and report to the physician any weight gain of10 lb.
B. Limit fluid intake to 500 mL/day.
C. Store the medication in a refrigerator and allow to stand at room temperature for 30 minutes prior to
administration.
D. Hold the vial under warm water for 10-15 minutes and shake vigorously before drawing medication
into the syringe.
Answer: D
Explanation:
(A) Weight should be obtained daily.
(B) Fluid is not restricted but is given according to urine output.
(C) The medication does not have to be stored in a refrigerator.
(D) Holding the vial under warm water for 10-15 minutes or rolling between your hands and shaking
vigorously before drawing medication into the syringe activates the medication in the oil solution.

Document Details

Related Documents

person
Harper Mitchell View profile
Close

Send listing report

highlight_off

You already reported this listing

The report is private and won't be shared with the owner

rotate_right
Close
rotate_right
Close

Send Message

image
Close

My favorites

image
Close

Application Form

image
Notifications visibility rotate_right Clear all Close close
image
image
arrow_left
arrow_right